Максимальное собственное число матрицы как найти

Математический портал

Nav view search

Navigation

Search

  • Вы здесь:
  • Home
  • Векторная алгебра.
  • Собственные числа и вектора матриц. Методы их нахождения

Собственные числа и вектора матриц. Методы их нахождения.

Литература: Сборник задач по математике. Часть 1. Под ред А. В. Ефимова, Б. П. Демидовича.

Пусть число $lambda$ и вектор $xin L, xneq 0$ таковы, что $$Ax=lambda x.qquadqquadqquadqquadqquad(1)$$ Тогда число $lambda$ называется собственным числом линейного оператора $A,$ а вектор $x$ собственным вектором этого оператора, соответствующим собственному числу $lambda.$

В конечномерном пространстве $L_n$ векторное равенство (1) эквивалентно матричному равенству $$(A-lambda E)X=0,,,,, Xneq 0.qquadqquadquadquad (2)$$

Отсюда следует, что число $lambda$ есть собственное число оператора $A$ в том и только том случае, когда детерминант $det(A-lambda E)=0,$ т. е. $lambda$ есть корень многочлена $p(lambda)=det(A-lambda E),$ называемого характеристическим многочленом оператора $A.$ Столбец координат $X$ любого собственного вектора соответствующего собственному числу $lambda$ есть нетривиальное решение однородной системы (2).

Примеры.

Найти собственные числа и собственные векторы линейных операторов, заданных своими матрицами.

Решение.

Найдем собственные вектора заданного линейного оператора. Число $lambda$ есть собственное число оператора $A$ в том и только том случае, когда $det(A-lambda E)=0.$ Запишем характеристическое уравнение:

$$det(A-lambda E)=begin2-lambda&-1&2\5&-3-lambda&3\-1&0&-2-lambdaend=$$ $$=(2-lambda)(-3-lambda)(-2-lambda)+3+2(-3-lambda)+5(-2-lambda)=$$ $$=-lambda^3-3lambda^2+4lambda+12+3-6-2lambda-10-5lambda=-lambda^3-3lambda^2-3lambda-1=0.$$

Решим найденное уравнение, чтобы найти собственные числа.

$$lambda^3+3lambda^2+3lambda+1=(lambda^3+1)+3lambda(lambda+1)=$$ $$=(lambda+1)(lambda^2-lambda+1)+3lambda(lambda+1)=(lambda+1)(lambda^2-lambda+1+3lambda)=$$ $$=(lambda+1)(lambda^2+2lambda+1)=(lambda+1)^3=0Rightarrow lambda=-1.$$

Собственный вектор для собственного числа $lambda=-1$ найдем из системы $$(A-lambda E)X=0, Xneq 0, Rightarrow (A+E)X=0, Xneq 0$$

Решим однородную систему уравнений:

Вычислим ранг матрицы коэффициентов $A=begin3&-1&2\5&-2&3\-1&0&-1end$ методом окаймляющих миноров:

Фиксируем минор отличный от нуля второго порядка $M_2=begin3&-1\5&-2end=-6+5=-1neq 0.$

Таким образом ранг матрицы $A$ равен двум.

Выберем в качестве базисного минор $M=begin3&-1\5&-2end=-1neq 0.$ Тогда, полагая $x_3=c,$ получаем: $$left<begin3x_1-x_2+2с=0\ 5x_1-2x_2+3с=0endright.Rightarrowleft<begin3x_1-x_2=-2c\5x_1-2x_2=-3cendright.$$

По правилу Крамера находим $x_1$ и $x_2:$

Таким образом, общее решение системы $X(c)=begin-c\-c\cend.$

Из общего решения находим фундаментальную систему решений: $E=X(1)=begin-1\-1\1end.$

С использованием фундаментальной системы решений, общее решение может быть записано в виде $X(c)=cE.$

Ответ: $lambda=-1;$ $X=cbegin-1\-1\1end, cneq 0.$

Решение.

Найдем собственные вектора заданного линейного оператора. Число $lambda$ есть собственное число оператора $A$ в том и только том случае, когда $det(A-lambda E)=0.$ Запишем характеристическое уравнение:

$$det(A-lambda E)=begin-lambda&-1&0\1&1-lambda&-2\1&-1&-lambdaend=$$ $$=-lambda(1-lambda)(-lambda)+2-lambda+2lambda=$$ $$=-lambda^3+lambda^2+lambda+2=0.$$

Решим найденное уравнение, чтобы найти собственные числа.

Собственный вектор для собственного числа $lambda=2$ найдем из системы $$(A-lambda E)X=0, Xneq 0, Rightarrow (A-2E)X=0, Xneq 0$$

Решим однородную систему уравнений:

Вычислим ранг матрицы коэффициентов $A=begin-2&-1&0\1&-1&-2\1&-1&-2end$ методом окаймляющих миноров:

Фиксируем минор отличный от нуля второго порядка $M_2=begin-2&-1\1&-1end=2+1=3neq 0.$

Таким образом ранг матрицы $A$ равен двум.

Выберем в качестве базисного минор $M=begin-2&-1\1&-1end=3neq 0.$ Тогда, полагая $x_3=c,$ получаем: $$left<begin-2x_1-x_2=0\ x_1-x_2-2с=0endright.Rightarrowleft<begin-2x_1-x_2=0\x_1-x_2=2cendright.$$

По правилу Крамера находим $x_1$ и $x_2:$

Таким образом, общее решение системы $X(c)=beginfrac<2c><3>\-frac<4c><3>\cend.$

Из общего решения находим фундаментальную систему решений: $E=X(1)=beginfrac<2><3>\-frac<4><3>\1end.$

С использованием фундаментальной системы решений, общее решение может быть записано в виде $X(c)=cE.$ Переобозначив постоянную, $alpha=3c,$ получаем собственный вектор $X=alphabegin2\-4\3end, alphaneq 0.$

Домашнее задание.

Найти собственные числа и собственные векторы линейных операторов, заданных своими матрицами.

Ответ: $lambda=2;$ $X=c_1begin1\2\0end+c_2begin0\0\1end, $c_1$ и $ c_2$ не равны одновременно нулю.

VMath

Инструменты сайта

Основное

Навигация

Информация

Действия

Содержание

Характеристический полином, собственные числа, собственные векторы матрицы

В настоящем разделе $ n_<> $ означает порядок квадратной матрицы $ A_<> $.

Характеристический полином

определяется для произвольной квадратной матрицы $ A_<> $ как 1) $ det (A_<>-lambda E) $, где $ E_<> $ – единичная матрица одинакового с $ A_<> $ порядка.

Пример. Для $ n=2_<> $:

Теорема 1.

Пример. Характеристический полином матрицы Фробениуса

$$ mathfrak F= left( begin 0 & 1 & 0 & 0 & dots & 0 & 0 \ 0 & 0 & 1 & 0 & dots & 0 & 0 \ 0 & 0 & 0 & 1 & dots & 0 & 0 \ vdots& &&&ddots & & vdots \ 0 & 0 & 0 & 0 & dots & 0 & 1 \ a_n & a_ & a_ & & dots & a_2 & a_1 end right)_ $$ равен $ (-1)^n(lambda^n-a_1lambda^-dots-a_) $.

Характеристический полином линейного оператора

определяется как характеристический полином матрицы этого оператора в произвольном базисе линейного пространства, в котором этот оператор задан. Подробнее ☞ ЗДЕСЬ.

Характеристический полином линейного однородного разностного уравнения

$ n_<> $-го порядка $$ x_=a_1 x_+ dots+ a_n x_K, quad a_n ne 0, $$ определяется как $$ lambda^n — a_1 lambda^ — dots — a_n . $$ Подробнее ☞ ЗДЕСЬ.

Свойства

Теорема 2. Характеристический полином матрицы не меняется

1. при ее транспонировании: $$ det (A-lambda E) = det (A^<top>-lambda E_<>) , ;$$ 2. при переходе к подобной матрице: если $ B=C^<-1>AC^<> $ при произвольной неособенной матрице $ C_<> $, то $$ det (A-lambda E) equiv det (B-lambda E_<>) , . $$

Теорема 3. Пусть матрица $ A_<> $ имеет порядок $ mtimes n_<> $, а $ B_<> $ — порядок $ ntimes m_<> $. Тогда эти матрицы допускают умножение в любом порядке, т.е. определены $ AB_<> $ и $ BA_<> $ и оба произведения будут квадратными матрицами — порядков $ m_<> $ и $ n_<> $ соответственно. Тогда характеристические полиномы этих произведений различаются лишь на степень $ lambda_<> $:

$$ lambda^n det (AB — lambda E_)equiv lambda^m det (BA — lambda E_) . $$

Если матрицы $ A_<> $ и $ B_<> $ — квадратные одинакового порядка, то характеристические полиномы матриц $ AB_<> $ и $ BA_<> $ тождественны.

Теорема 4. Если характеристический полином матрицы $ A_<> $ равен

$$ f(lambda)=(-1)^n lambda^n+a_1lambda^+dots+a_lambda+a_n $$ и $ a_ ne 0 $, то характеристический полином матрицы $ A^<-1>_<> $ равен $$ f^<ast>(lambda)=frac<(-lambda)^n> f(1/lambda) = frac<(-1)^n> left[ (-1)^n+a_1 lambda + dots+ a_lambda^+a_nlambda^ right] . $$

Теорема Гамильтона-Кэли

Теорема 5. Результатом подстановки в характеристический полином $ det (A_<>-lambda E) $ самой матрицы $ A_<> $ будет нулевая матрица:

$$ det (A-lambda E)= (-1)^n lambda^n +a_1 lambda^+dots+a_lambda+ a_n Rightarrow $$ $$ Rightarrow (-1)^n A^n +a_1 A^+dots+a_A+ a_n E = <mathbb O>_ . $$

матрица является корнем своего характеристического полинома.

Доказательство ☞ ЗДЕСЬ.

Пример. Для $ n_<>=2 $:

$$ left(begin a_ <11>& a_ <12>\ a_ <21>& a_ <22>end right)^2 — (a_<11>+a_<22>)left(begin a_ <11>& a_ <12>\ a_ <21>& a_ <22>end right) + (a_<11>a_<22>-a_<12>a_<21>) left(begin 1 & 0 \ 0 & 1 end right) = left(begin 0 & 0 \ 0 & 0 end right) . $$

Собственное число

определяется для квадратной матрицы $ A_<> $ как произвольный корень ее характеристического полинома $ det (A_<>-lambda E) $. Набор всех собственных чисел матрицы $ A_<> $ (с учетом их кратностей) называется спектром матрицы (таким образом спектр матрицы $ A_<> $ порядка $ n_<> $ всегда состоит из $ n_<> $ чисел, часть из которых могут быть одинаковыми). Максимальный из модулей собственных чисел матрицы $ A_<> $ называется ее спектральным радиусом, он иногда обозначается $ rho(A) $.

Пример. Найти спектр матрицы

$$ A= left(begin 0&1&2&3\ -1&0&4&7\ -2&-4&0&2\ -3&-7&-2&0 endright). $$ Решение. Характеристический полином $$ det (A-lambda E)=left|begin -lambda&1&2&3\ -1&-lambda&4&7\ -2&-4&-lambda&2\ -3&-7&-2&-lambda endright|=lambda^4+ 83lambda^2 $$ имеет корни $ lambda_1=0, lambda_2 = <mathbf i>sqrt<83>, lambda_3 = — <mathbf i>sqrt <83>$, причем $ lambda_ <1>$ — второй кратности.

Ответ. Спектр матрицы $ A_<> $: $ <0,0, <mathbf i>sqrt<83>,- <mathbf i>sqrt <83>> $. Спектральный радиус матрицы $ A_<> $: $ rho(A)= sqrt <83>$.

Теорема 6. Если $ <lambda_<1>,lambda_<2>,dots,lambda_ > $ — спектр матрицы $ A_<> $, то

$$ lambda_1+lambda_<2>+dots+lambda_n = operatorname(A)=a_<11>+a_<22>+dots+a_, $$ $$ lambda_1cdotlambda_<2>times dots times lambda_n = (-1)^ndet (A) . $$

Доказательство следует из представления характеристического полинома через миноры матрицы и формул Виета. ♦

Для того, чтобы матрица $ A_<> $ была неособенной необходимо и достаточно, чтобы среди ее собственных чисел не было нулевого.

Теорема 7. Пусть $ g(x)=b_<0>x^m+dots+b_m in <mathbb C>[x] $ — произвольный полином. Вычислим полином от матрицы $ A_<> $: $ g(A)=b_<0>A^m+dots+b_m E $. Тогда если $ <lambda_<1>,dots,lambda_ > $ — спектр матрицы $ A_<> $, то $ ),dots,g(lambda_n) > $ — спектр матрицы $ g(A_<>) $.

Результат теоремы обобщается и на более широкий класс функций $ g_<>(x) $ — фактически на любую функцию, которая может быть определена на спектре матрицы $ A_<> $. В частности, если $ det A_<> ne 0 $, то спектр матрицы $ A^<-1>_<> $ совпадает с $ <1/lambda_j>_^n $.

Имеет место следующее равенство, связывающее степени матрицы $ A_<> $ с суммами Ньютона ее характеристического полинома:

$$ operatorname(A^k)=lambda_1^k+dots+lambda_n^k . $$ Здесь $ operatorname_<> $ обозначает след матрицы (т.е. сумму ее диагональных элементов). Утверждение остается справедливым и для отрицательных показателей $ k_<> $ при условии, что $ det A_<> ne 0 $.

Имеет место следующее равенство:

$$ det g(A) = (-1)^ <mathcal R>(f,g_<>) , $$ где $ <mathcal R>(f,g_<>) $ означает результант полиномов $ f(x) =det (A-x_<> E) $ и $ g_<>(x) $.

Теорема 8. Собственные числа вещественной симметричной матрицы $ A_<> $ все вещественны.

Доказательство ☞ ЗДЕСЬ.

Теорема 9. Собственные числа вещественной кососимметричной матрицы $ A_<> $ все мнимы, за исключением, возможно, $ lambda_<> = 0 $.

Доказательство ☞ ЗДЕСЬ.

Теорема 10. Собственные числа вещественной ортогональной матрицы все равны $ 1_<> $ по абсолютной величине (модулю). Характеристический полином ортогональной матрицы является возвратным если $ +1 $ не является его корнем или является корнем четной кратности. Хотя бы одно собственное число ортогональной матрицы нечетного порядка равно $ +1 $ или $ (-1) $.

Доказательство ☞ ЗДЕСЬ.

Теорема 11. Спектр циклической матрицы

$$ left(begin a_1 & a_2 & a_3 & dots & a_n \ a_n & a_1 & a_2 & dots & a_ \ a_ & a_n & a_1 & dots & a_ \ vdots & & & & vdots \ a_2 & a_3 & a_4 & dots & a_1 end right) . $$ совпадает с набором чисел $$ ) > ,$$ при $$ f(x)=a_<1>+a_2x+a_3x^2+dots+a_nx^ $$ и $$ varepsilon_k=cos frac<2,pi k> + <mathbf i>sin frac<2,pi k> $$ — корне n-й степени из единицы.

Доказательство ☞ ЗДЕСЬ.

Локализация собственных чисел

Теорема 12. [1]. Собственные числа матрицы являются непрерывными функциями ее элементов. Иначе: пусть

$$A=left[a_ right]_^n quad , quad B=left[b_ right]_^n . $$ Обозначим $$M= max_> left <|a_|, |b_ | right> quad , quad delta = frac<1>sum_^n |a_ — b_ | . $$ Тогда любому собственному числу $ lambda_<ast>^<> $ матрицы $ A_<> $ можно поставить в соответствие такое собственное число $ mu_<ast>^<> $ матрицы $ B_<> $, что $$ |lambda_<ast>-mu_ <ast>| le (n+2) M sqrt[n] <delta> . $$

Собственно факт непрерывной зависимости собственных чисел от элементов матрицы следует из представления характеристического полинома из теоремы ☞ ПУНКТА — коэффициенты этого полинома полиномиально (и, следовательно, непрерывно) зависят от элементов матрицы. Далее используем теорему о непрерывной зависимости корней полинома от его коэффициентов.

Выясним теперь на примере, насколько малым может быть возмущение элементов матрицы чтобы сохранились хотя бы количество вещественных корней ее характеристического полинома.

Пример [Уилкинсон] [2]. Найти собственные числа матрицы

$$ A= left( begin 20 & 20 & & & & \ & 19 & 20 & & & \ & & 18 & 20 & & \ & & & ddots & ddots & \ & & & & 2 & 20 \ <colorvarepsilon > & & & & & 1 \ end right)_ <20times 20>$$ при $ <colorvarepsilon >=10^ <-10>$ (все неуказанные элементы матрицы считаются равными нулю).

Решение. Характеристический полином $$ det(A-lambda E) = prod_^ <20>(j-lambda) — 20^ <19> <colorvarepsilon > = $$ $$ =lambda^<20>-<scriptstyle 210>,lambda^<19>+<scriptstyle 20615>,lambda^<18>-<scriptstyle 1256850>, lambda^ <17>+<scriptstyle 53327946>, lambda^<16>-<scriptstyle 1672280820>, lambda^<15>+ <scriptstyle 40171771630>, lambda^<14>-<scriptstyle 756111184500>, lambda^<13>+ $$ $$ +<scriptstyle 11310276995381>, lambda^ <12>- <scriptstyle 135585182899530>, lambda^ <11>+<scriptstyle 1307535010540395>, lambda^<10>-<scriptstyle 10142299865511450>, lambda^9 + $$ $$ +<scriptstyle 63030812099294896>, lambda^8 — <scriptstyle 311333643161390640>, lambda^7+<scriptstyle 1206647803780373360>, lambda^6 -<scriptstyle 3599979517947607200>, lambda^5 +<scriptstyle 8037811822645051776>, lambda^4- $$ $$ -<scriptstyle 12870931245150988800>, lambda^3 +<scriptstyle 13803759753640704000>, lambda^2 -<scriptstyle 8752948036761600000>,lambda + <scriptstyle 2432377720176640000>$$ очень похож на полином из другого ☞ ПРИМЕРА Уилкинсона. Он имеет корни $$ lambda_1=0.995754, lambda_2=2.109241, lambda_3=2.574881, $$ $$ lambda_<4,5>=3.965331pm 1.087735, mathbf i, lambda_<6,7>=5.893977pm 1.948530 , mathbf i, $$ $$ lambda_<8,9>=8.118073 pm 2.529182 , mathbf i, lambda_<10,11>=10.5pm 2.733397 , mathbf i, $$ $$ lambda_<12,13>=12.881926pm 2.529182 , mathbf i, lambda_<14,15>=15.106022 pm 1.948530 , mathbf i, $$ $$ lambda_<16,17>=17.034669pm 1.087735 , mathbf i, $$ $$ lambda_<18>=18.425118, lambda_<19>=18.890758, lambda_<20>=20.004245 . $$ Итак, нановозмущение 2) в одном-единственном элементе матрицы приводит к существенному изменению спектра: из $ 20 $ вещественных собственных чисел «остаются в живых» только $ 6_<> $; кроме того, у образовавшихся мнимых корней оказываются достаточно большими мнимые части. В данном примере допустимые возмущения для $ <colorvarepsilon > $, т.е. такие, при которых сохранится свойство вещественности всех корней характеристического полинома, находятся в пределах 3) $$ -8.636174times 10^<-14> ♦

Теорема 13 [Гершгорин]. 4) Обозначим $ mathbb D_ $ круг на комплексной плоскости $ mathbb C_<> $ с центром в точке $ a_^<> $ и радиуса

$$ r_j=sum_<ell=1 atop ellne j>^n left|a_right| .$$ Тогда спектр матрицы $ A_<> $ лежит внутри объединения этих кругов: $$ <lambda_1,dots, lambda_n >subset bigcup_^n mathbb D_j . $$ Иными словами: любое собственное число матрицы должно удовлетворять хотя бы одному из неравенств $$ |z- a_ |

Если все главные миноры $ A_1,A_2,dots,A_ $ симметричной матрицы $ A_<> $ отличны от нуля, то число положительных собственных чисел матрицы $ A_<> $ равно числу знакопостоянств, а число отрицательных собственных чисел — числу знакоперемен в ряду $ 1,A_1,dots,A_n $:

$$ operatorname < det (A-lambda E) =0 | lambda>0 > = <mathcal P>(1,A_1,dots,A_n), $$ $$ operatorname < det (A-lambda E) =0 | lambda ненулевойстолбец $$ X_<ast>= left( begin x_<1>^ <ast>\ vdots \ x_^ <ast>end right) in mathbb^n $$ такой, что $$ AX_<ast>=lambda_ <ast>X_ <ast>quad iff quad (A -lambda_<ast>E) X_ <ast>= mathbb O_ . $$ По определению собственного числа, $ det (A^<> -lambda_<ast>E) = 0 $ и, следовательно, система однородных уравнений $ (A -lambda_<ast>E) X^<> = mathbb O $ всегда имеет нетривиальное решение; более того, этих решений бесконечно много. Таким образом, одному и тому же собственному числу матрицы принадлежит бесконечное множество собственных векторов. Эту бесконечность можно описать с помощью фундаментальной системы решений (ФСР).

Пример. Найти собственные векторы матрицы

Решение. Спектр матрицы найден выше. $$(A-0 cdot E)X=mathbb O quad Longrightarrow mbox< ФСР>= left< <mathfrak X>_1=left(begin 4 \ -2 \ 1 \ 0 endright), <mathfrak X>_2=left(begin 7 \ -3 \ 0 \ 1 end right) right>.$$ Любой вектор вида $ alpha_ <1><mathfrak X>_1 + alpha_2 <mathfrak X>_2 $ будет собственным, принадлежащим $ lambda_<>=0 $. $$ begin (A- mathbf i, sqrt <83>E)X=mathbb O \ \ Downarrow \ \ <mathfrak X>_3= left(begin 1- mathbf i , sqrt <83>\ 8-2, mathbf i , sqrt <83>\ 12 \ 17+mathbf i , sqrt <83>endright) end qquad begin (A+mathbf i sqrt <83>E)X=mathbb O \ \ Downarrow \ \ <mathfrak X>_4= left(begin 1+mathbf i , sqrt <83>\ 8+2mathbf i , sqrt <83>\ 12 \ 17- mathbf i ,sqrt <83>endright) end . $$ ♦

Еще один способ нахождения собственного вектора основан на теореме Гамильтона-Кэли.

Теорема 15. Пусть $ lambda_<ast>^<> $ — собственное число матрицы $ A_<> $. Обозначим частное от деления характеристического полинома на линейный множитель $ lambda_<> — lambda_ <ast>$ через $ f_<ast>(lambda)^<> $:

$$ f_<ast>(lambda) equiv f(lambda) / (lambda-lambda_<ast>) . $$ Тогда любой ненулевой столбец матрицы $ f_<ast>(A)^<> $ является собственным вектором, принадлежащим $ lambda_<ast>^<> $.

Доказательство следует из равенства $$(A-lambda_ <ast>E)f_<ast>(A)=mathbb O_ . $$ На основании определения любой ненулевой столбец $ f_<ast>(A)^<> $ должен быть собственным вектором матрицы $ A_<> $. ♦

Пример. Найти собственные векторы матрицы

$$ A=left( begin 9 & 22 & -6 \ -1 &-4 & 1 \ 8 & 16 & -5 end right) . $$

Решение. $$ det (A-lambda E)=-lambda^3+ 7, lambda + 6 equiv -(lambda_<>-3) (lambda+2)(lambda+1) , .$$ Пренебрегая знаком – , имеем: $$ begin f_1(lambda)=lambda^2+3lambda+2 & u & f_1(A)= left( begin 40 & 80 & -20 \ 0 &0 & 0 \ 40 & 80 & -20 end right) , \ f_2(lambda)=lambda^2-2lambda-3 & u & f_2(A)= left( begin -10 & -30 & 10 \ 5 &15 & -5 \ 0 & 0 & 0 end right) , \ f_3(lambda)=lambda^2-lambda-6 & u & f_3(A)= left( begin -4 & -8 & 4 \ 4 & 8 & -4 \ 8 & 16 & -8 end right) . end $$

Если $ lambda_<ast>^<> $ является простым корнем характеристического полинома 5) , то ненулевые столбцы $ f_<ast>(A)^<> $ будут пропорциональными. Или, что то же, $ operatorname f_<ast>(A)^<> = 1 $.

Тогда очевидно, что и строки матрицы $ f_<ast>(A)^<> $ тоже должны быть пропорциональны!

Доказать, что любая ненулевая строка матрицы $ f_<ast>(A)^<> $ является собственным вектором матрицы $ A^<^<top>>_<> $, принадлежащим $ lambda_<ast>^<> $. Доказать, что собственный вектор матрицы $ A_<> $ ортогонален собственному вектору матрицы $ A^<top>_<> $, если эти векторы принадлежат различным собственным числам 6) .

На практике вычисление полинома $ f_<ast>(lambda)^<> $ может быть осуществлено с помощью схемы Хорнера.

Пример. Вычислить собственный вектор матрицы

$$ A=left( begin 23 & 75 & -92 \ 6 & 74 & 72 \ 37 & -23 & 87 end right) , $$ принадлежащий ее вещественному собственному числу.

Решение. Характеристический полином $$ f(lambda)= -lambda^3+184,lambda^2-14751,lambda+611404 $$ имеет единственное вещественное собственное число $ lambda_ <ast>approx 96.8817 $. Составляем схему Хорнера $$ begin & -1 & 184 & -14751 & 611404 \ hline 96.8817 & -1 & 87.1183 & -6310.8310 & -0.0352 end $$ За счет ошибок округления мы получили ненулевое значение для $ f(lambda_<ast>) $. В качестве частного от деления $ f(lambda) $ на $ lambda-lambda_ <ast>$ берем $$ f_<ast>(lambda)= -lambda^2 + 87.1183, lambda — 6310.8310 . $$ Подставляем в него матрицу $ A_<> $ и вычисляем первый столбец матрицы $$ -A^2+87.1183,A -6310, E = left( begin -1882.1101 & * & * \ -2723.2902 & * & * \ -708.6229 & * & * end right) .$$ Проверяем: $$ left( begin 23 & 75 & -92 \ 6 & 74 & 72 \ 37 & -23 & 87 end right) left( begin -1882.1101 \ -2723.2902 \ -708.6229 end right) — 96.8817 left( begin -1882.1101 \ -2723.2902 \ -708.6229 end right)= left( begin 0.0356 \ 0 \ -0.0002 end right) . $$ ♦

Можно развить последний метод далее: найти универсальную формулу для собственного вектора как функции ее собственного числа. Действительно, найдем частное от деления характеристического полинома $$ f(lambda) =a_0lambda^n+a_0lambda^+dots+ a_n, quad a_0=(-1)^n $$ на линейный полином $ lambda- lambda_ <ast>$, где $ lambda_ <ast>$ — произвольное число из $ mathbb C $. С помощью той же схемы Хорнера, получаем $$ q(lambda)=a_0lambda^+(a_0lambda_<ast>+a_1)lambda^+(a_0lambda_<ast>^2+a_1lambda_<ast>+a_2)lambda^+dots+ (a_0lambda_<ast>^+a_1lambda_<ast>^+dots+a_) , . $$ Если $ lambda_ <ast>$ является собственным числом матрицы $ A_<> $, то любой ненулевой столбец матрицы $$ q(A)= a_0A^+(a_0lambda_<ast>+a_1)A^+(a_0lambda_<ast>^2+a_1lambda_<ast>+a_2)A^+dots+ (a_0lambda_<ast>^+a_1lambda_<ast>^+dots+a_)E $$ будет собственным вектором, принадлежащим $ lambda_ <ast>$.

Пример. Найти представление всех собственных векторов матрицы

$$ A=left( begin 9 & 22 & -6 \ -1 &-4 & 1 \ 8 & 16 & -5 end right) $$ в виде функции ее собственных чисел.

Решение. Характеристический полином матрицы был вычислен выше: $ f(lambda)=-lambda^3+ 7, lambda + 6 $. Имеем, $$ q(lambda)=-lambda^2-lambda_<ast>lambda+(7-lambda_<ast>^2) $$ и $$ q(A)=-A^2-lambda_<ast>A+(7-lambda_<ast>^2)E= left(begin -lambda_<ast>^2-9lambda_<ast>-4 & -22lambda_<ast>-14 & 6lambda_<ast>+2 \ lambda_<ast>-3 & -lambda_<ast>^2+4lambda_<ast>-3 & -lambda_<ast>+3 \ -8lambda_<ast>-16 & -16lambda_<ast>-32 & -lambda_<ast>^2+5lambda_<ast>+14 end right) , . $$ Берем произвольный столбец этой матрицы, например, первый: $$ X_<ast>(lambda_<ast>)= left(begin -lambda_<ast>^2-9lambda_<ast>-4 \ lambda_<ast>-3 \ -8lambda_<ast>-16 end right) , . $$ Утверждается, что $ X_ <ast>(lambda_<ast>) $ — универсальное представление всех собственных векторов матрицы. Действительно, $$ X_<ast>(-1) = left(begin 4 \ -4 \ -8 end right), X_<ast>(-2) = left(begin 10 \ -5 \ 0 end right), X_<ast>(3) = left(begin -40 \ 0 \ -40 end right) , . $$ ♦

Теорема 16. Пусть $ g(x)=b_0x^m+dots+b_ in <mathbb C>[x] $ – произвольный полином. Если $ X_<ast>in mathbb C^ $ — собственный вектор матрицы $ A_<> $, соответствующий собственному числу $ lambda_<ast>^<> $, то он же будет собственным и для матрицы $ g(A)^<> $, принадлежащим собственному числу $ g(lambda_<ast>)^<> $.

Доказательство. Домножим равенство $ A<mathfrak X>_<ast>=lambda_<ast>^<><mathfrak X>_ <ast>$ слева на матрицу $ A_<> $: $$ A^2<mathfrak X>_<ast>=lambda_<ast>A<mathfrak X>_<ast>=lambda_<ast>^2<mathfrak X>_ <ast>.$$ По индукции доказывается и общее равенство: $$ A^k<mathfrak X>_<ast>=lambda_<ast>^k<mathfrak X>_ <ast>.$$ Домножим его на $ b_^<> $ и просуммируем по $ k_<> $ от $ 0_<> $ до $ m_<> $: $$ g(A)<mathfrak X>_<ast>=g(lambda_<ast>)<mathfrak X>_ <ast>,$$ что и доказывает утверждение теоремы. ♦

Если матрица $ A $ невырождена, то теорема остается справедливой и для произвольного полинома от $ A^ <-1>$. В частности, собственные векторы $ A^ <-1>$ совпадают с собственными векторами матрицы $ A $.

Теорема 17. Собственные векторы, принадлежащие различным собственным числам матрицы $ A_<> $, линейно независимы.

Теорема 18. Собственные векторы, принадлежащие различным собственным числам вещественной симметричной матрицы $ A_<> $, ортогональны, т.е. если $ mathfrak X_1 $ принадлежит собственному числу $ lambda_ <1>$, а $ mathfrak X_2 $ принадлежит собственному числу $ lambda_ <2>$ и $ lambda_1 ne lambda_2 $, то

$$ langle mathfrak X_1, mathfrak X_2 rangle =0 , $$ где $ langle , rangle $ означает скалярное произведение, определяемое стандартным образом: $ langle X,Y rangle =x_1y_1+dots+x_ny_n $.

Доказательство ☞ ЗДЕСЬ.

Теорема Перрона-Фробениуса

Теорема 19 [Перрон, Фробениус]. Для положительной матрицы $ A_<> $ существует положительное собственное число $ lambda_ <+>$ такое, что все остальные собственные числа этой матрицы меньше $ lambda_ <+>$ по абсолютной величине (модулю). Соответствующий этому собственному числу собственный вектор может быть выбран положительным:

$$ exists mathfrak X_ <+>> mathbb O: quad A mathfrak X_ <+>= lambda_ <+>mathfrak X_ <+> . $$

Число $ lambda_ <+>$ из теоремы называется собственным числом Перрона или собственным числом Перрона-Фробениуса матрицы $ A_<> $, а соответствующий ему произвольный положительный собственный вектор — собственным вектором Перрона-Фробениуса матрицы $ A_<> $.

Спектральный радиус положительной матрицы $ A_<> $ совпадает с ее собственным числом Перрона-Фробениуса:

Пример. Найти собственное число и вектор Перрона-Фробениуса для матрицы

$$ A= left(begin 2 & 7 & 18 & 28 \ 1 & 8 & 2 & 8 \ 3 & 1 & 4 & 1 \ 5 & 9 & 26 & 5 end right) , . $$

Решение. Характеристический полином матрицы $ A_<> $ $$ det(A-lambda E)=lambda^4-19, lambda^3-175, lambda^2-285, lambda+10390 $$ имеет корнями $$ lambda_ <1,2>approx -6.260463 pm 5.452465 mathbf i, lambda_3 approx 5.878976, lambda_4 approx 25.641950 . $$ Числом Перрона-Фробениуса является $ lambda_4 $, а соответствующий ему собственный вектор Перрона-Фробениуса можно взять равным $$ left( begin 1 \ 0.365240 \ 0.184802 \ 0.634244 end right) quad mbox < или >quad left( begin 2.737922 \ 1 \ 0.505974 \ 1.736510 end right) quad mbox < или >left( begin 5.411185 \ 1.976383 \ 1 \ 3.432010 end right) quad mbox < или >quad left( begin 1.576681 \ 0.575868 \ 0.291374 \ 1 end right) quad mbox < или >quad left( begin 0.798133 \ 0.291510 \ 0.147496\ 0.506210 end right) quad mbox < или >dots $$ (напоминаю: собственный вектор определяется с точностью до ненулевого сомножителя!). Последний вектор имеет длину равную $ 1_<> $. ♦

1. Собственное число Перрона-Фробениуса всегда простое для характеристического полинома матрицы $ A_<> $. Отсюда следует, что собственный вектор Перрона-Фробениуса определяется единственным образом — с точностью до домножения на положительный скаляр.

2. Любой собственный вектор положительной матрицы $ A_<> $, не соответствующий собственному числу Перрона-Фробениуса, не может состоять исключительно только из положительных элементов. Иными словами, хотя бы одна компонента такого вектора должна быть либо отрицательной либо мнимой.

3. Для собственного числа Перрона-Фробениуса справедливо неравенство $$ min_> sum_^n a_ le lambda_ <+>le max_> sum_^n a_ . $$

4. Собственное число Перрона-Фробениуса матрицы $ A_<> $ совпадает с собственным числом Перрона-Фробениуса матрицы $ A^ <top>$.

Какие из перечисленных свойств можно распространить на случай неотрицательных матриц ? Каждую такую матрицу можно рассматривать как предел последовательности (строго) положительных матриц. Воспользовавшись теоремой о непрерывной зависимости собственных чисел матрицы от ее элементов, можем сделать вывод, о том, что для неотрицательной матрицы $ A_<> $ всегда найдется вещественное неотрицательное собственное число, которое будет являться максимальным по модулю среди всех собственных чисел матрицы. Другое дело, что в данном случае — в отличие от случая положительных матриц — такое мажорирующее собственное число может оказаться не единственным.

Пример. Спектр неотрицательной матрицы

$$ A=left( begin 0 & 1 \ 1 & 0 end right) $$ состоит из чисел $ lambda_1=+1 $ и $ lambda_1=-1 $ одинакового модуля. ♦

Однако, по-прежнему, хотя бы одно неотрицательное вещественное число $ lambda_ <+>$ со свойством $ rho(A) = lambda_ <+>$ существовать будет; более того, ему будет соответствовать неотрицательный собственный вектор $ mathfrak X ge mathbb O $. Это число (вектор) по-прежнему называются числом (вектором) Перрона-Фробениуса 7) матрицы $ A_<> $.

Частным случаем неотрицательных матриц являются стохастические матрицы, т.е. неотрицательные матрицы, в которых сумма элементов каждой строки равна $ 1_<> $: $$ P=left[p_right]_^n, \ge 0 >_^n, sum_^n p_ = 1 npu quad j in <1,2,dots,n> . $$

Теорема 20. Собственное число Перрона-Фробениуса стохастической матрицы равно $ 1_<> $. Этому собственному числу соответствует собственный вектор $ X=[1,1,dots,1]^ <top>$.

Доказательство существования собственного числа равного $ 1_<> $ и соответствующего ему собственного вектора $ X=[1,1,dots,1]^ <top>$ следует из равенства $$ P left( begin 1 \ 1 \ vdots \ 1 end right) = left( begin 1 \ 1 \ vdots \ 1 endright) . $$ Далее, из теоремы Гершгорина следует, что любое собственное число $ lambda_<>in mathbb C $ стохастической матрицы должно удовлетворять неравенству $$|lambda — p_|le sum_ |p_|=1-p_ $$ хотя бы при одном $ j_<> $. Воспользовавшись следствием к неравенству треугольника получаем: $$|lambda| — |p_|le |lambda — p_| le 1-p_ Rightarrow |lambda| le 1 . $$ ♦

Методы вычисления характеристического полинома

Вычисление коэффициентов характеристического полинома матрицы $ A_<> $ непосредственным разложением определителя $ det (A-lambda_<> E) $ на $ n!_<> $ слагаемых — крайне неэффективно. Элементами этого разложения являются выражения, полиномиально зависящие от параметра $ lambda_<> $. На каждом этапе вычислений мы получаем проблему символьных вычислений: хранения таких полиномов и действий над ними.

Основной метод вычисления числовых определителей — метод Гаусса — также неэффективен в приложении к вычислению определителя, элементы которого зависят от параметра. Источником вычислительных проблем является неудобное расположение переменной $ lambda_<> $ — на главной диагонали матрицы. Первый же шаг метода Гаусса приводит к делению на элемент $ a_ <11>- lambda $, и, в дальнейшем, элементы преобразованной матрицы будут уже не полиномами, а рациональными функциями относительно $ lambda_<> $. Следующие шаги метода приводят к возрастанию степеней знаменателей. Необходимость в организации хранения рациональных функций и программировании действий с ними кажется тем более неоправданной, если вспомнить, что окончательный ответ — выражение для $ det (A-lambda_<> E) $ — должно быть полиномом по $ lambda_<> $; т.е. знаменатели дробей в конечном ответе сократятся.

А в качестве усугубляющего положение обстоятельства «на заднем плане» маячит проблема точности вычислений коэффициентов характеристического полинома — чувствительность его корней к возмущению его коэффициентов бывает весьма высокой.

Какой выход предлагается? — Предварительно преобразовать определитель $ det (A-lambda_<> E) $ к виду, когда переменная $ lambda_<> $ оказывается «выметенной» с диагонали на крайний ряд (в столбец или в строку). При этом допускается увеличение размеров (порядка) определителя. Такое представление дает возможность разложения определителя по этому исключительному ряду, и, тем самым, позволяет свести задачу к вычислению числовых определителей — а уж для этой задачи применение метода Гаусса вполне эффективно.

Метод Леверье

Метод основан на формуле (см. следствие к теореме $ 7 $ ☞ ЗДЕСЬ ): $$ operatorname (A^k)=lambda_1^k+dots+lambda_n^k=s_k , $$ т.е. след $ k_<> $-й степени матрицы $ A_<> $ равен $ k_<> $-й сумме Ньютона ее характеристического полинома $ f(lambda)=det (A-lambda E ) $. Вычисляем последовательные степени матрицы $ A_<> $: $$s_1=operatorname (A), s_2=operatorname (A^2), dots, s_n=operatorname (A^n) .$$ Неизвестные коэффициенты $ f(lambda)=(-1)^n(lambda^n+a_1lambda^+ dots+a_n) $ находим по рекурсивным формулам Ньютона: $$ a_1=-s_1, a_2=-(s_2+a_1s_1)/2, $$ $$ a_k=-(s_+a_1s_+a_2s_+dots+a_s_1)/k npu k le n. $$ Очевидно, что не имеет смысла вычислять все элементы матрицы $ A^ $ — достаточно обойтись лишь элементами ее главной диагонали.

Пример [Леверье]. Найти характеристический полином матрицы

$$ A=left(begin -5.509882&1.870086&0.422908&0.008814 \ 0.287865&-11.811654&5.711900&0.058717 \ 0.049099&4.308033&-12.970687&0.229326 \ 0.006235&0.269851&1.397369&-17.596207 end right) . $$

Решение. $$ A^2=left(begin 30.91795128&-30.56848188&2.878480155&0.0031325713\ -4.705449283&164.6764010&-141.3504639&-0.4143169528\ 0.3341843103&-106.6094396&193.1869924& -6.756396001\ 0.0022236138&-1.904168948&-41.16923134& 309.9628536 end right), $$ $$ A^3=left(begin -179.0125092&431.2849919&-198.8601505& -0.9173897610\ 66.38829278&-2562.954533& 2771.458834& -15.49709921\ -23.08728044&2090.291485&-3124.010318& 156.9329019\ -0.649145142&-71.21907809&956.2502143& -5463.723497 end right), $$ $$ A^4=left(begin 1100.720103& ast& ast& ast \ ast& 42332.23816& ast& ast \ ast& ast& 52669.62534& ast \ ast& ast& ast& 96355.91518 end right) . $$ Вычисляем следы матриц: $$s_1=-47.888430, s_2=698.7441983, s_3=-11329.70086, s_4= 192458.4988 ,$$ и по формулам Ньютона получаем: $$a_1= 47.888430, a_2 = 797.278764_ <displaystyle 8>, a_3 = 5349.45551_<displaystyle 3>, a_4 = 12296.550_ <displaystyle 68> . $$ ♦

После нахождения коэффициентов характеристического полинома можно найти его корни каким-либо 8) методом. Если $ lambda_<ast>^<> $ — одно из собственных чисел, то для нахождения соответствующего собственного вектора воспользуемся алгоритмом из ☞ ПУНКТА. Предположив дополнительно, что это собственное число простое 9) , обозначим $$ f_<ast>(lambda)= f(lambda)/(lambda-lambda_<ast>)=(-1)^n(lambda^ +p_1lambda^+dots+p_lambda+p_) $$ т.е. частное от деления $ f(lambda_<>) $ на $ lambda-lambda_ <ast>$. Тогда любой ненулевой столбец матрицы $ f_<ast>(A)^<> $ будет собственным вектором, принадлежащим $ lambda_<ast>^<> $. Как правило, собственным вектором можно взять комбинацию

Степени матрицы $ A_<> $ уже нами посчитаны при вычислении коэффициентов характеристического полинома.

Пример. Для приведенного выше примера находим собственные числа:

$$ lambda_1=-17.86326, lambda_2=-17.15242, lambda_3=-7.57404, lambda_4= -5.29869 . $$ Коэффициенты $ f_1(lambda) $ можно определить по схеме Хорнера: $$ begin &1 & 47.888430 & 797.2787648 & 5349.455513 & 12296.55068 \ hline -17.86326 & 1 & underbrace<30.025170>_>& underbrace<260.9313465>_> &underbrace<688.371028>_>& approx 0 \ end $$ Собственным вектором, принадлежащим $ lambda_ <1>$, будет $$left[ -0.0256_<displaystyle 67>, 0.21938_<displaystyle 0>, -0.24187_<displaystyle 1>, 1.044526 right]^<^<top>> .$$ ♦

Теорема 21. Характеристический полином явно выражается через суммы Ньютона с помощью следующего представления:

$$ f(lambda)=frac<1>left| begin s_1 &1 & & & &\ s_2&s_1& 2 & &mathbb O & \ s_3&s_2&s_1&3& & \ vdots& & & ddots &ddots & \ s_n&s_& s_ & dots &s_1&n \ lambda^n&lambda^&lambda^& dots &lambda&1 end right|_ <(n+1)times (n+1)> . $$

Биографические заметки о Леверье ☞ ЗДЕСЬ.

Метод Крылова

Рассмотрим произвольный ненулевой столбец $ Y_0=left[ y_<1>^<[0]>,dots,y_^ <[0]>right]^<^<top>> in mathbb C^n $. Cоставим итерационную векторную последовательность $$ Y_1=Acdot Y_0, Y_2=Acdot Y_1, dots, Y_=Acdot Y_ . $$

Теорема 22. Определитель

$$ det left[begin Y_0&Y_<1>&dots&Y_&Y_\ 1& lambda&dots&lambda^&lambda^n end right]_ <(n+1)times (n+1)>$$ совпадает — с точностью до постоянного множителя — с характеристическим полиномом матрицы $ A_<> $. Здесь $ |_<> $ означает конкатенацию.

Доказательство. Легко видеть, что $$ Y_K=A^KY_0 quad npu quad K in <1,dots,n> . $$ Если $$ f(lambda)=det(A-lambda E) =(-1)^n lambda^n+a_1 lambda^+a_2 lambda^+dots+a_n , $$ то по теореме Гамильтона-Кэли: $$ (-1)^n A^n+a_1A^+dots+a_nE=mathbb O_ . $$ Это равенство останется справедливым и после умножения его на произвольный вектор, в том числе на $ Y_ <0>$: $$ (-1)^n A^ncdot Y_0+a_1A^ cdot Y_0 +dots+a_ncdot Y_0=mathbb O_ iff $$ $$ iff quad (-1)^n Y_n+a_1Y_ +dots+a_nY_0=mathbb O . $$ Последнее равенство представляет линейную систему относительно неизвестных коэффициентов характеристического полинома. Можно решать ее по формулам Крамера, но мы пойдем другим путем. Дополним эту систему тождеством $ f(lambda)=(-1)^n lambda^n+a_1 lambda^+a_2 lambda^+dots+a_n $. Рассмотрим получившуюся систему как линейную однородную относительно столбца $ left[ a_n,a_,dots,a_1,1right]^ <top>$. Поскольку эта система имеет нетривиальное решение, то ее определитель должен равняться нулю: $$ 0=det left[begin Y_0&Y_<1>&dots&Y_&(-1)^nY_\ 1& lambda&dots&lambda^&(-1)^nlambda^n-f(lambda) end right]= $$ (представляем последний столбец в виде суммы двух столбцов и используем свойство 5 определителя) $$ =det left[begin Y_0&Y_<1>&dots&Y_&(-1)^nY_\ 1& lambda&dots&lambda^&(-1)^nlambda^n end right]-f(lambda) det left[begin Y_0&Y_<1>&dots&Y_ end right] . $$ Таким образом, $$ f(lambda)=(-1)^n frac<det left[begin Y_0&Y_<1>&dots&Y_&Y_\ 1& lambda&dots&lambda^&lambda^n end right]><det left[begin Y_0&Y_<1>&dots&Y_ end right]> , $$ если только знаменатель в этой дроби не обратится в нуль. ♦

Пример. Найти характеристический полином матрицы примера Леверье

$$ A=left(begin -5.509882&1.870086&0.422908&0.008814 \ 0.287865&-11.811654&5.711900&0.058717 \ 0.049099&4.308033&-12.970687&0.229326 \ 0.006235&0.269851&1.397369&-17.596207 end right) . $$

Решение. Возьмем $ Y_0=left[ 1,0,0,0 right]^ <top>$. Имеем $$ begin Y_1=A Y_0= & Y_2=AY_1= & Y_3=AY_2= & Y_4=AY_3= \ left(begin -5.509882\ 0.287865 \ 0.049099 \ 0.006235 end right), & left(begin 30.917951\ -4.705449 \ 0.334184 \ 0.002223 end right), & left(begin -179.012509\ 66.388293 \ -23.087280\ -0.649145 end right), & left(begin 1100.720101\ -967.597333\ 576.522644\ -4.040153 end right) . end $$ $$ det left[begin Y_0&Y_<1>&Y_2& Y_<3>& Y_<4>\ 1& lambda&lambda^2 &lambda^<3>&lambda^4 end right]= left| begin 1 & -5.509882 & 30.917951 & -179.012509 & 1100.720101 \ 0 & 0.287865 & -4.705449 & 66.388293 & -967.597333\ 0 & 0.049099 & 0.334184 & -23.087280 & 576.522644\ 0 & 0.006235 & 0.002223 & -0.649145 & -4.040153 \ 1 & lambda & lambda^2 & lambda^3 & lambda^4 end right|= $$ $$ =0.348621 lambda^4+16.694915lambda^3+277.948166lambda^2+1864.932835lambda+4286.836454 = $$ $$ =0.348621 left(lambda^4+47.888430lambda^3+797.27876_<displaystyle 3>lambda^2+5349.4555_<displaystyle 0>lambda+12296.550_ <displaystyle 5>right) . $$ ♦

После нахождения характеристического полинома можно найти его корни каким-либо 10) методом. Пусть $ lambda_<ast>^<> $ — одно из собственных чисел, и оно — простое; тогда для нахождения соответствующего собственного вектора можно воспользоваться тем же приемом, что был задействован в предыдущем ПУНКТЕ. Вычислим 11) частное от деления $ f(lambda_<>) $ на $ lambda-lambda_ <ast>$ $$ f_<ast>(lambda)= f(lambda)/(lambda-lambda_<ast>)=(-1)^n(lambda^ +p_1lambda^+dots+p_lambda+p_) . $$ Тогда любой ненулевой столбец матрицы $ f_<ast>(A)^<> $ будет собственным вектором, принадлежащим $ lambda_<ast>^<> $. Но тогда и произвольная комбинация столбцов этой матрицы тоже будет собственным вектором (если только не обратится в нулевой вектор). В частности, это относится и к комбинации, записываемой в матричном виде $$ (-1)^n f_<ast>(A) Y_0 = A^Y_0 +p_1A^Y_0+dots+p_Y_0=Y_+p_1Y_+dots+p_Y_0 . $$ А комбинируемые векторы уже посчитаны.

Теперь обсудим исключительные случаи. При неудачном выборе $ Y_ <0>$ определитель $$ det left[begin Y_0&Y_<1>&dots&Y_ end right] $$ может обратиться в нуль. Эта неприятность обязательно произойдет если, например, наш выбор пал на вектор $ Y_0 $, совпадающий с собственным вектором матрицы $ A_<> $. Вероятность такого события — нулевая. В общем же случае, трудно ожидать, чтобы $ n_<> $ почти произвольных столбцов $ Y_0,Y_<1>,dots,Y_ $ оказались линейно зависимыми — если только сама матрица $ A_<> $ не обладает «скрытым дефектом» — типа рассмотренного в следующем примере.

Пример. Найти характеристический полином матрицы

Решение. При любом выборе $ Y_0 $ векторы $ $ оказываются линейно зависимыми: $$ Y_0= left(begin 1\ 0\ 0 end right), Y_1= left(begin 2\ 1\ 1 end right), Y_2= left(begin 6\ 5\ 5 end right),dots ; Y_0= left(begin 1\ 1\ 1 end right), Y_1= left(begin 4\ 4\ 4 end right),dots $$ Объяснение этого феномена состоит в том, что для матрицы $ A_<> $ ее аннулирующий полином имеет степень меньшую ее порядка: $$ A^2-5 A+4 E = mathbb O . $$ Домножение этого равенства на произвольный столбец $ Y_0 $ и доказывает линейную зависимость системы $ $. ♦

Такая ситуация возможна только в случае, когда характеристический полином матрицы $ A_<> $ имеет кратные корни (в рассмотренном выше примере $ lambda_<>=1 $ являлся двойным корнем $ det (A-lambda_<> E) $); она исключительно редко встречается на практике.

Поиск всех собственных чисел

Существуют методы нахождения спектра матрицы, не требующие предварительного построения характеристического полинома.

QR-алгоритм

Этот алгоритм основан на QR-разложении матрицы $ A $.

Теорема 23. Спектр матрицы $ A $ совпадает со спектром матрицы $ P^ <top>A P $ при произвольной ортогональной матрице $ P $.

Доказательство. $$ det (P^ <top>A P-lambda E)=det (P^ <top>A P- lambda P^ <top>E P)=det P^ <top>(A -lambda E ) P = det (A -lambda E ) P P^ <top>= det (A -lambda E ) , . $$ ♦

Пусть QR-разложение матрицы $ A $ имеет вид $$ A=Q_1R_1 , , $$ где $ Q_1 $ — ортогональная, а $ R_1 $ — верхнетреугольная матрицы. Тогда матрица $$ A_2=R_1Q_1 $$ имеет тот же спектр, что и матрица $ A $. Действительно, поскольку $$ A_2=Q_1^ <top>A Q_1 , $$ то сработает предыдущая теорема. Вычислим QR-разложение матрицы $ A_2 $ $$ A_2=Q_2R_2 $$ и переставим местами матрицы этого произведения: $$ A_3=R_2Q_2 , . $$ Матрица $$ A_3= Q_2^ <top>A_2 Q_2=Q_2^ <top>Q_1^ <top>A Q_1 Q_2 $$ продолжаем иметь те же собственные числа, что и матрица $ A $. Утверждается, что бесконечная последовательность матриц $$ Q_>_^ <infty>$$ как правило, сходится к матрице $ A_ <infty>$, которая будет верхнетреугольной.

Теорема 24 [4]. Если все собственные числа матрицы $ A $ различны по модулю, то матрица $ A_ <infty>$ является верхнетреугольной и на ее главной диагонали стоят собственные числа матрицы $ A $.

Пример. Найти все собственные числа матрицы $$ A=left(begin 2 & 3 &-1\ 7 & 3 & 3 \ -1 & -2 & 4 end right) , . $$

Решение. $$ A_1=Aapprox underbrace<left(begin 0.272165 & 0.759752 & 0.590511 \ 0.952579 & -0.299517 & -0.053683 \ -0.136083& -0.577119 & 0.805242 end right)>_ underbrace<left(begin 7.348469 & 3.946400 & 2.041241\ 0 & 2.534941 & -3.966781 \ 0 & 0 & 2.469409 end right)>_ $$ Теперь переставляем матрицы произведения местами и строим QR-разложение получившейся матрицы: $$ quad Rightarrow quad A_2 = R_1Q_1approx left(begin 5.481481 & 3.222957 & 5.771191 \ 2.954542 & 1.530046 & -3.3303021 \ -0.336044 & -1.425143 & 1.988472 end right)approx $$ $$ approxunderbrace<left(begin -0.878992 & 0.022595 & 0.476300\ 0.473781 & -0.154267 & -0.867026 \ 0.053886 & -0.987771 & 0.146304 end right)>_ underbrace<left(begin -6.236096& -3.634658 & -3.387848\ 0 & 1.244502 & -1.319999\ 0 & 0 & 5.927198 end right)>_ $$ Продолжим процесс: $$ quad Rightarrow quad A_3 = R_2Q_2approx left(begin 7.020952& 3.766220 & -0.314568\ -0.660752 & 1.111870 & -1.272137\ 0.319398 & -5.854713 & 0.867177 end right) approx $$ $$ approx underbrace<left(begin -0.994581 & -0.065879 & 0.080426 \ 0.093601 & -0.230749 & 0.968501 \ -0.045246 & 0.970780 & 0.235665 end right)>_ underbrace<left(begin -7.059205 & -3.376839 & 0.154554 \ 0 & -6.188319 & 1.156106 \ 0 & 0 & -1.053002 end right)>_ $$ Замечаем тенденцию убывания элементов матриц $ $, стоящих под главной диагональю. $$ Rightarrow dots Rightarrow A_ <10>approx left(begin mathbf<6.>_ <246022>& 2.758769 & -2.160057\ -0.0467437 & mathbf<4.4>_ <09292>& -5.341014\ 0.000018 &-0.005924 & mathbf<-1.6>_ <55314>end right) approx $$ $$ underbrace<left(begin -0.999972 & -0.007483 & 0.000007 \ 0.007483 & -0.999971 & 0.001339 \ -0.000003 & 0.001339 & 0.999999 end right)>_> underbrace<left(begin -6.246197 & -2.725694 & 2.120031\ 0 & -4.429817 & 5.354807 \ 0 & 0 & -1.662479 end right)>_> , . $$ Матрица $ Q_j $ уже близка к диагональной (с элементами $ pm 1 $), верхнетреугольность матрицы $ A_j $ также заметна, но точность приближения еще не достаточна. $$ Rightarrow dots Rightarrow A_ <20>approx left(begin mathbf<6.17>_ <5608>& 2.805821 & -2.020513 \ -0.001776 & mathbf<4.48>_ <4917>& -5.388407\ 0 & 0 & -mathbf <1.660525>end right) approx $$ Точность приближения минимильного собственного числа существенно выше точностей приближения остальных чисел. $$ Rightarrow dots Rightarrow A_ <30>approx left(begin mathbf<6.172>_ <778>& 2.807524 & -2.015076\ -0.000073 & mathbf<4.487>_ <747>& -5.390442\ 0 & 0 & -mathbf <1.660525>end right) , . $$ ♦

К сожалению условие теоремы достаточно ограничительно: собственные числа вещественной матрицы $ A $ могут оказаться и мнимыми, но тогда они одинаковы по модулю.

Как это обстоятельство сказывается на структуре матрицы $ A_ <infty>$ и дальнейшее развитие метода ☞ ЗДЕСЬ

Частичная проблема собственных чисел

Задача. Найти максимальное по модулю собственное число матрицы $ A_<> $.

Предположение . Будем считать сначала, что максимальное по модулю собственное число матрицы единственно.

Излагаемый ниже метод поиска этого собственного числа называется методом степенны́х итераций 12) .

Рассмотрим произвольный ненулевой столбец $ Y_0=left[ y_<1>^<[0]>,dots,y_^ <[0]>right]^<^<top>> in mathbb C^n $. Cоставим такую же итерационную векторную последовательность, как и в методе Крылова $$ Y_1=Acdot Y_0, Y_2=Acdot Y_1, dots, Y_=Acdot Y_,dots , $$ (только теперь, в отличие от метода Крылова, считаем ее неограниченно продолжающейся) и выделим последовательность первых элементов этих векторов: $$y_<1>^<[1]>,y_<1>^<[2]>,dots,y_<1>^<[K]>,dots $$

Теорема 25. Как правило, предел

$$ lim_frac^<[K+1]>>^<[K]>> $$ существует и он равен максимальному по модулю собственному числу матрицы $ A_<> $.

Доказательство. Перенумеруем собственные числа $ lambda_<1>,dots,lambda_n $ матрицы $ A_<> $ так, чтобы $ lambda_ <1>$ обозначило максимальное по модулю: $$|lambda_1|= max_> |lambda_j| , quad |lambda_1|>|lambda_j| quad npu quad jin <2,dots,n> . $$ Очевидно, $$ Y_=A^Kcdot Y_0 ; $$ отсюда следует, что любой элемент столбца $ Y_ $ может быть линейно выражен через $ lambda_<1>^K,dots,lambda_n^K $. В частности, это справедливо и для первого элемента: $$ y_<1>^<[K]>=C_1lambda_1^K+C_2lambda_2^K+dots+C_nlambda_n^K . $$ В этом представлении $ _^n $ — будут константами из $ mathbb C_<> $ в случае если все собственные числа являются простыми, и полиномами из $ mathbb C[K] $ в случае, если имеются кратные собственные числа. Действительно, в первом случае существует базис пространства $ mathbb C^n $, состоящий из собственных векторов матрицы $ A_<> $: $$ A<mathfrak X>_j=lambda_j<mathfrak X>_j quad npu quad jin <1,dots,n>. $$ Вектор $ Y_0 $ можно разложить по этому базису: $$Y_0=alpha_1<mathfrak X>_1+dots+alpha_n<mathfrak X>_n .$$ Тогда последовательным домножением на матрицу $ A_<> $ получаем : $$begin Y_1=AY_0&=& alpha_1 lambda_1<mathfrak X>_1+dots+alpha_nlambda_n<mathfrak X>_n, \ dots & & dots \ Y_K=A^KY_0&=& alpha_1 lambda_1^K<mathfrak X>_1+dots+alpha_nlambda_n^K<mathfrak X>_n end $$ откуда и следует доказываемое равенство.

Во втором случае — когда имеются кратные собственные числа матрицы $ A_<> $ — придется применять «тяжелую артиллерию» в виде жордановой нормальной формы; см. теорему $ 5 $ ☞ ЗДЕСЬ. Для простоты рассуждений, будем в оставшейся части доказательства считать все собственные числа матрицы различными. Имеем тогда $$ lim_ frac^<[K+1]>>^<[K]>>= lim_ frac <lambda_1^left[C_1+ C_2(lambda_2/lambda_1)^+dots+ C_n(lambda_n/lambda_1)^ right]> <lambda_1^left[C_1+C_2(lambda_2/lambda_1)^+dots+ C_n(lambda_n/lambda_1)^ right]> =lambda_1 $$ поскольку $$ lim_ left| frac<lambda_j> <lambda_1>right|^K = 0 quad npu quad jin <2,dots,n> . $$ Исключительным случаем является ситуация $ C_1=0 $, в этом случае утверждение теоремы может оказаться несправедливым 13) . ♦

Как правило, вектор

$$ left[1, lim_frac^<[K]>>^<[K]>>,dots, lim_frac^<[K]>>^<[K]>>right]^<^<top>> $$ будет собственным, принадлежащим максимальному по модулю собственному числу матрицы $ A_<> $.

Пример. Для матрицы

$$ A=left(begin 2 & 3 &-1\ 7 & 3 & 3 \ -1 & -2 & -4 end right) $$ найти максимальное по модулю собственное число и принадлежащий ему собственный вектор.

Решение. Возьмем в качестве стартового столбца $ Y_0=[1,0,0]^<^<top>> $. Имеем: $$ Y_1=AY_0=left( begin 2 \ 7 \ -1 end right), Y_2=AY_1=left( begin 26 \ 32 \ -12 end right), Y_3=AY_2=left( begin 160 \ 242 \ -42 end right),dots, $$ $$ Y_<19>=left( begin <scriptstyle 4259667747238636>\ <scriptstyle 6435097324667832>\ <scriptstyle -1571397155909260>end right), Y_<20>=AY_<19>=left( begin <scriptstyle 29396024624390028>\ <scriptstyle 44408774736946168>\ <scriptstyle -10844273772937260>end right) $$ Смотрим на отношения первых элементов векторов: $$ begin K & 1 & 2 & 3 & 4 & 5 & dots & 15 & dots & 19 \ hline y_<1>^<[K+1]>/y_<1>^ <[K]>& 2 & 13 & 6.153846 & 6.8 & 7.180147 & dots & 6.900726 & dots & mathbf<6.90101>_ <displaystyle 3>end $$ Далее, в соответствии со следствием, собственный вектор, принадлежащий найденному числу $$ approx left[1, frac^<[20]>>^<[20]>>,frac^<[20]>>^<[20]>>right]^<^<top>> approx left[1, 1.51070_<displaystyle 6>, -0.368902 right]^<^<top>> $$ ♦

Теперь обсудим исключительные случаи алгоритма.

1. Нарушение сходимости итерационного процесса за счет неудачного выбора стартового вектора. Если в качестве $ Y_ <0>$ оказался случайно взят собственный вектор $ mathfrak X_ <ast>$ матрицы $ A_<> $, принадлежащий произвольному ее собственному числу $ lambda_ <*>$, то предел последовательности из теоремы будет равен именно этому числу; если при этом $ |lambda_ <*>| ne max_ <1le j le n>| lambda_j | $, то мы выйдем за пределы смысла выражения «как правило». Понятно, что вероятность настолько плохого выбора нулевая, но и выбор $ Y_0 $ вблизи $ mathfrak X_ <ast>$ также может существенно замедлить скорость сходимости. Поэтому если возникает ситуация медленной «стабилизации» значащих цифр в десятичном приближении собственного числа, попробуйте сменить начальный вектор.

2. Нарушение условия предположения , выдвинутого в начале пункта: максимальное по модулю собственное число неединственно.

Пример. Найти максимальное по модулю собственное число матрицы примера Леверье

$$ A=left(begin -5.509882&1.870086&0.422908&0.008814 \ 0.287865&-11.811654&5.711900&0.058717 \ 0.049099&4.308033&-12.970687&0.229326 \ 0.006235&0.269851&1.397369&-17.596207 end right) . $$

Решение. Для столбца $ Y_0=[1,0,0,0]^<^<top>> $ имеем $$y_<1>^<[100]>/y_<1>^<[99]>=-17.8_ <displaystyle 3113> ,$$ т.е. на $ 100 $-й итерации получаем лишь $ 3_<> $ истинные десятичные цифры в представлении собственного числа. При этом компонентами векторов $ Y_ $ являются числа порядка $ 10^ <123>$. Если мы посмотрим на ответ примера Леверье, то увидим, что имеются два собственных числа матрицы, близких по модулю. ♦

К сожалению, вероятность того факта, что у случайно выбранной матрицы два ее собственных числа будут иметь одинаковый модуль становится ненулевой если эта матрица выбирается из множества вещественных матриц. Дело в том, что в этом случае ее характеристический полином будет иметь вещественные коэффициенты, а мнимые корни такого полинома всегда пáрные — для любого невещественного корня $ lambda_<ast>^<> $ полинома, комплексно сопряженное к нему число $ overline<lambda_<ast>> $ также будет корнем. При этом $ |lambda_<ast>|= |overline<lambda_<ast>> | $.

Пример. Для матрицы

Предположение 2 . Пусть два максимальных по модулю собственных числа матрицы разнесены по величине, например $$ |lambda_1| > | lambda_2 | > | lambda_ j | quad npu j in <2,dots, n >. $$

Обобщение степенного метода основывается на использовании последовательностей из каких-то двух компонент векторов $ Y_=AY_K $, например, наряду с уже использованной выше последовательностью первых компонент $$y_<1>^<[1]>,y_<1>^<[2]>,dots,y_<1>^<[K]>,dots $$ возьмем еще и аналогичную для вторых: $$y_<2>^<[1]>,y_<2>^<[2]>,dots,y_<2>^<[K]>,dots $$

Теорема 26 [Эйткен]. При практически любом выборе стартового вектора $ Y_0 ne mathbb O $ для последовательности

Доказательство. Построим квадратное уравнение $$ p_0x^2+p_1x+p_2 = 0 $$ имеющее корнями $ lambda_1 $ и $ lambda_2 $. Если существует базис рпостранства $ mathbb C^n $ $$Y_0=alpha_1<mathfrak X>_1+alpha_2<mathfrak X>_2+dots+alpha_n<mathfrak X>_n .$$ Тогда последовательным домножением на матрицу $ A_<> $ получаем : $$begin Y_K=& alpha_1 lambda_1^K<mathfrak X>_1 &+alpha_2 lambda_2^K<mathfrak X>_2+dots &+alpha_nlambda_n^K<mathfrak X>_n, \ Y_=& alpha_1 lambda_1^<mathfrak X>_1 &+alpha_2 lambda_2^<mathfrak X>_2+dots &+alpha_nlambda_n^<mathfrak X>_n,\ Y_=& alpha_1 lambda_1^<mathfrak X>_1 & +alpha_2 lambda_2^<mathfrak X>_2+dots &+alpha_nlambda_n^<mathfrak X>_n. end $$ Отбрасываем из правых частей равенств слагаемые порядков возрастания ниже, чем $ lambda_2^K, lambda_2^, lambda_2^ $ соответственно, домножаем получившиеся приближенные равенства $$begin Y_K & approx alpha_1 lambda_1^K<mathfrak X>_1 &+alpha_2 lambda_2^K<mathfrak X>_2, & color times p_2 \ Y_& approx alpha_1 lambda_1^<mathfrak X>_1 &+alpha_2 lambda_2^<mathfrak X>_2, & color times p_1\ Y_ & approx alpha_1 lambda_1^<mathfrak X>_1 & +alpha_2 lambda_2^<mathfrak X>_2, & color times p_0 end $$ и складываем: $$ p_2 Y_K + p_1Y_ + p_0 Y_ approx mathbb O , . $$ В получившемся векторном равенстве выбираем первые две компоненты: $$ left< begin p_2 y_1^ <[K]>+ p_1 y_1^ <[K+1]>+ p_0 y_1^ <[K+2]>approx 0 , , \ p_2 y_2^ <[K]>+ p_1 y_2^ <[K+1]>+ p_0 y_2^ <[K+2]>approx 0 , , end right. $$ которые и позволят определить приближенное значение набора $ p_0,p_1,p_2 $. С точностью до числового сомножителя, искомый полином можно представить в виде определителя $$ p_0x^2+p_1x+p_2 approx left|begin y_1^ <[K]>& y_1^ <[K+1]>& y_1^ <[K+2]>\ y_2^ <[K]>& y_2^ <[K+1]>& y_2^ <[K+2]>\ 1 & x & x^2 end right| , . $$ Формулы Виета завершат доказательство. ♦

При выполнении условия предположения 2 имеет место равенство

Пример. Для матрицы

$$ A=left(begin 2 & 3 &-1\ 7 & 3 & 3 \ -1 & -2 & 4 end right) $$ найти первые два по порядку убывания модулей собственных числа.

Задачи

Источники

[2]. Уилкинсон Дж.Х. Алгебраическая проблема собственных значений. М.Наука. 1970, с.93-94

[3]. Фаддеев Д.К., Фаддеева В.Н. Вычислительные методы линейной алгебры. М.ГИФМЛ. 1960

[4]. Хорн Р., Джонсон Ч. Матричный анализ. М.Мир.1989

источники:

http://vmath.ru/vf5/algebra2/charpoly

Пусть


,

т.е.

и

– собственное значение и собственный
вектор матрицы A.

Умножая обе части данного равенства на

,
получим


.

Следовательно,

– собственный вектор матрицы

,
а

– собственное значение матрицы

.

3.4.6. Обобщенная проблема собственных значений.

Пусть даны матрицы

и

,
при этом

– невырожденная. Требуется найти
ненулевой вектор

и число , удовлетворяющие
равенству


.
(3.4.23)

В этом случае

и

,
соответственно, называются обобщенными
собственным числом
и собственным
вектором
.

Умножая на

обе части (3.4.23), получим


.
(3.4.24)

Таким образом, задачу можно свести к
определению собственных чисел и векторов
матрицы

.

На практике в современных стандартных
программных комплексах реализована
непосредственно обобщенная проблема
собственных значений. Для этих целей,
например, используется так называемый
QZ-алгоритм, представляющий
собой устойчивое обобщение QR-алгоритма.

3.4.7. Вычисление минимального собственного числа степенным методом.

Как уже отмечалось в п. 3.4.5. собственные
векторы матриц

и

совпадают, а собственные значения
связаны соотношением


,
(3.4.25)

откуда

,
(3.4.26)

где

– минимальное собственное значение
матрицы

;

– максимальное собственное значение
матрицы

.

Таким образом, для определения минимального
собственного числа матрицы

достаточно вычислить максимальное
собственное число матрицы

степенным методом.

§ 3.5. Методы численного интегрирования

3.5.1. Понятие о формулах численного интегрирования.

Пусть требуется вычислить определенный
интеграл вида


.
(3.5.1)

Для многих функций

первообразные представляют собой
достаточно сложные комбинации элементарных
функций, либо вовсе не выражаются через
них. В таких случаях использование
формулы Ньютона-Лейбница на практике
не представляется возможным. Во многих
практических случаях достаточно получить
значение интеграла с заданной точностью

.
Для вычисления приближенного значения
интеграла существуют формулы численного
интегрирования. Суть построения формул
численного интегрирования состоит в
следующем.

Разобьем отрезок

на

частей. Для простоты изложения положим
эти части одинаковой длины

.

Пронумеруем точки разбиения так, как
показано на рис. 3.5.1. Имеем:


,

при этом

.

Рис. 3.5.1. К вопросу
о численном интегрировании.

Исходный интеграл (3.5.1) может быть
представлен в виде суммы интегралов по
полученным в результате разбиения
«малым» отрезкам:


.
(3.5.2)

Интегралы

(3.5.3)

вычисляются по приближенным формулам.

Простейшие формулы для приближенного
вычисления интегралов по отрезку
называются квадратурными формулами
[8,66]. Рассмотрим некоторые из них ниже,
а также изучим вопросы их точности.
Порядок точности квадратурной формулы
определяется степенью полинома
(многочлена), для которой эта квадратурная
формула точна.

Соседние файлы в предмете [НЕСОРТИРОВАННОЕ]

  • #
  • #
  • #
  • #
  • #
  • #
  • #
  • #
  • #
  • #
  • #

Методы решения задач о собственных
значениях и векторах матриц

Постановка задачи

Пусть [math]A[/math] — действительная числовая квадратная матрица размера [math](ntimes n)[/math]. Ненулевой вектор [math]X= bigl(x_1,ldots,x_nbigr)^T[/math] размера [math](ntimes1)[/math], удовлетворяющий условию

[math]Acdot X= lambdacdot X,qquad mathsf{(2.1)}[/math]

называется собственным вектором матрицы [math]A[/math]. Число [math]lambda[/math] в равенстве (2.1) называется собственным значением. Говорят, что собственный вектор [math]X[/math] соответствует (принадлежит) собственному значению [math]lambda[/math].

Равенство (2.1) равносильно однородной относительно [math]X[/math] системе:

[math](A-lambda E)cdot X=0quad (Xne 0).qquad mathsf{(2.2)}[/math]

Система (2.2) имеет ненулевое решение для вектора [math]X[/math] (при известном [math]lambda[/math]) при условии [math]|A-lambda E|=0[/math]. Это равенство есть характеристическое уравнение:

[math]|A-lambda E|= P_n(lambda)=0,[/math]

(2.3)

где [math]P_n(lambda)[/math] — характеристический многочлен n-й степени. Корни [math]lambda_1, lambda_2,ldots,lambda_n[/math] характеристического уравнения (2.3) являются собственными (характеристическими) значениями матрицы [math]A[/math], а соответствующие каждому собственному значению [math]lambda_i,~ i=1,ldots,n[/math], ненулевые векторы [math]X^i[/math], удовлетворяющие системе

[math]AX^i=lambda_iX^iquad text{or}quad (A-lambda_i E)X^i=0,~~ i=1,2,ldots,n,[/math]

(2.4)

являются собственными векторами.

Требуется найти собственные значения и собственные векторы заданной матрицы. Поставленная задача часто именуется второй задачей линейной алгебры.

Проблема собственных значений (частот) возникает при анализе поведения мостов, зданий, летательных аппаратов и других конструкций, характеризующихся малыми смещениями от положения равновесия, а также при анализе устойчивости численных схем. Характеристическое уравнение вместе с его собственными значениями и собственными векторами является основным в теории механических или электрических колебаний на макроскопическом или микроскопическом
уровнях.

Различают полную и частичную проблему собственных значений, когда необходимо найти весь спектр (все собственные значения) и собственные векторы либо часть спектра, например: [math]rho(A)= max_{i}|lambda_i(A)|[/math] и [math]min_{i}|lambda_i(A)|[/math]. Величина [math]rho(A)[/math] называется спектральным радиусом.

Замечания

1. Если для собственного значения [math]lambda_i[/math] — найден собственный вектор [math]X^i[/math], то вектор [math]mu X^i[/math], где [math]mu[/math] — произвольное число, также является собственным вектором, соответствующим этому же собственному значению [math]lambda_i[/math].

2. Попарно различным собственным значениям соответствуют линейно независимые собственные векторы; k-кратному корню характеристического уравнения соответствует не более [math]k[/math] линейно независимых собственных векторов.

3. Симметрическая матрица имеет полный спектр [math]lambda_i,~ i=overline{1,n}[/math], действительных собственных значений; k-кратному корню характеристического уравнения симметрической матрицы соответствует ровно [math]k[/math] линейно независимых собственных векторов.

4. Положительно определенная симметрическая матрица имеет полный спектр действительных положительных собственных значений.


Метод непосредственного развертывания

Полную проблему собственных значений для матриц невысокого порядка [math](nleqslant10)[/math] можно решить методом непосредственного развертывания. В этом случае будем иметь

[math]|A-lambda E|= begin{vmatrix}a_{11}-lambda& a_{12}& a_{13}& cdots& a_{1n}\ a_{21}& a_{22}-lambda& a_{23}& cdots& a_{2n}\ vdots& vdots& vdots& ddots& vdots\ a_{n1}& a_{n2}& a_{n3}& cdots& a_{nn}-lambda end{vmatrix}= P_n(lambda)=0.[/math]

(2.5)

Уравнение [math]P_n(lambda)=0[/math] является нелинейным (методы его решения изложены в следующем разделе). Его решение дает [math]n[/math], вообще говоря, комплексных собственных значений [math]lambda_1,lambda_2,ldots,lambda_n[/math], при которых [math]P_n(lambda_i)=0~ (i=overline{1,n})[/math]. Для каждого [math]lambda_i[/math] может быть найдено решение однородной системы [math](A-lambda_iE)X^i=0,~ i=overline{1,n}[/math]. Эти решения [math]X^i[/math], определенные с точностью до произвольной константы, образуют систему [math]n[/math], вообще говоря, различных векторов n-мерного пространства. В некоторых задачах несколько этих векторов (или все) могут совпадать.

Алгоритм метода непосредственного развертывания

1. Для заданной матрицы [math]A[/math] составить характеристическое уравнение (2.5): [math]|A-lambda E|=0[/math]. Для развертывания детерминанта [math]|A-lambda E|[/math] можно использовать различные методы, например метод Крылова, метод Данилевского или другие методы.

2. Решить характеристическое уравнение и найти собственные значения [math]lambda_1, lambda_2, ldots,lambda_n[/math]. Для этого можно применить методы, изложенные далее.

3. Для каждого собственного значения составить систему (2.4):

[math](A-lambda_iE)cdot X^i=0,quad i=1,2,ldots,n[/math]

и найти собственные векторы [math]X^i[/math].

Замечание. Каждому собственному значению соответствует один или несколько векторов. Поскольку определитель [math]|A-lambda_iE|[/math] системы равен нулю, то ранг матрицы системы меньше числа неизвестных: [math]operatorname{rang}(A-lambda_iE)=r<n[/math] и в системе имеется ровно [math]r[/math] независимых уравнений, а [math](n-r)[/math] уравнений являются зависимыми. Для нахождения решения системы следует выбрать [math]r[/math] уравнений с [math]r[/math] неизвестными так, чтобы определитель составленной системы был отличен от нуля. Остальные [math](n-r)[/math] неизвестных следует перенести в правую часть и считать параметрами. Придавая параметрам различные значения, можно получить различные решения системы. Для простоты, как правило, попеременно полагают значение одного параметра равным 1, а остальные равными 0.

Пример 2.1. Найти собственные значения и собственные векторы матрицы [math]Ain mathbb{R}^{2times 2}[/math], где [math]A=begin{pmatrix}3&-2\-4&1end{pmatrix}[/math].

Решение

Воспользуемся методикой.

1. Запишем уравнение (2.5): [math]|A-lambda E|= begin{vmatrix}3-lambda&-2\-4& 1-lambda end{vmatrix}= lambda^2-4 lambda-5=0[/math], отсюда получаем характеристическое уравнение [math]P_2(lambda)equiv lambda^2-4 lambda-5=0[/math].

2. Находим его корни (собственные значения): [math]lambda_1=5,~ lambda_2=-1[/math].

3. Составим систему [math](A-lambda_iE)X^i=0,~ i=1,2[/math], для каждого собственного
значения и найдем собственные векторы:

[math]begin{pmatrix}3-lambda_1&-2\-4& 1-lambda_1 end{pmatrix}! cdot! begin{pmatrix}x_1^1\x_2^1end{pmatrix}=0[/math] или [math]begin{cases}-2x_1^1-2x_2^1=0,\-4x_1^1-4x_2^1=0.end{cases}[/math]

Отсюда [math]x_1^1=-x_2^1[/math]. Если [math]x_2^1=mu[/math], то [math]x_1^1=-mu[/math]. В результате получаем [math]X^1= bigl{x_1^1, x_2^1bigr}^T= bigl{mu(-1;1)bigr}^T[/math].

Для [math]lambda_2=-1[/math] имеем

[math]begin{pmatrix}3-lambda_2&-2\-4& 1-lambda_2 end{pmatrix}! cdot! begin{pmatrix}x_1^2\x_2^2end{pmatrix}=0[/math] или [math]begin{cases}4x_1^2-2x_2^2=0,\-4x_1^2+2x_2^2=0.end{cases}[/math]

Отсюда [math]x_2^2=2x_1^2[/math]. Если [math]x_1^2=mu[/math], то [math]x_2^2=2mu[/math]. В результате получаем [math]X^2= bigl{x_1^2, x_2^2bigr}^T= bigl{mu(1;2)bigr}^T[/math], где [math]mu[/math] — произвольное действительное число.

Пример 2.2. Найти собственные значения и собственные векторы матрицы [math]A= begin{pmatrix}2&-1&1\-1&2&-1\0&0&1end{pmatrix}[/math].

Решение

Воспользуемся методикой.

1. Запишем характеристическое уравнение (2.5):

[math]begin{vmatrix}2-lambda&-1&1\-1&2-lambda&-1\0&0&1-lambda end{vmatrix}=0[/math] или [math](1-lambda)bigl[(2-lambda)^2-1bigr]=0[/math].

2. Корни характеристического уравнения: [math]lambda_{1,2}=1[/math] (кратный корень), [math]lambda_3=3[/math] — собственные значения матрицы.

3. Найдем собственные векторы.

Для [math]lambda_{1,2}=1[/math] запишем систему [math](A-lambda_{1,2}E)cdot X^{1,2}=0colon[/math]

[math]begin{pmatrix}1&-1&1\-1&1&-1\ 0&0&0end{pmatrix}! cdot! begin{pmatrix} x_1^{1,2}\ x_2^{1,2}\ x_3^{1,2}end{pmatrix}=0.[/math]

Поскольку [math]operatorname{rang}(A-lambda_{1,2}E)=1[/math], в системе имеется одно независимое уравнение

[math]x_1^{1,2}-x_2^{1,2}+x_3^{1,2}=0[/math] или [math]x_1^{1,2}=x_2^{1,2}-x_3^{1,2}[/math].

Полагая [math]x_2^{1,2}=1,~ x_3^{1,2}=3[/math], получаем [math]x_1^{1,2}=1[/math] и собственный вектор [math]X^1= begin{pmatrix}1&1&0end{pmatrix}^T[/math].

Полагая [math]x_2^{1,2}=0,~ x_3^{1,2}=1[/math], получаем [math]x_1^{1,2}=-1[/math] и другой собственный вектор [math]X^2= begin{pmatrix}-1&0&1end{pmatrix}^T[/math]. Заметим, что оба собственных вектора линейно независимы.

Для собственного значения [math]lambda_3=3[/math] запишем систему [math](A-lambda_3E)cdot X^3=0colon[/math]

[math]begin{pmatrix}-1&-1&1\-1&-1&-1\ 0&0&-2end{pmatrix}!cdot! begin{pmatrix} x_1^3\ x_2^3\ x_3^3end{pmatrix}=0.[/math]

Поскольку [math]operatorname{rang}(A-lambda_3E)=2[/math], то выбираем два уравнения:

[math]-x_1^3-x_2^3+x_3^3=0,qquad-2x_3^3=0.[/math]

Отсюда [math]x_3^3=0,~ x_1^3=-x_2^3[/math]. Полагая [math]x_2^3=1[/math], получаем [math]x_1^3=-1[/math] и собственный вектор [math]X^3=begin{pmatrix}-1&1&0 end{pmatrix}^T[/math].


Метод итераций для нахождения собственных значений и векторов

Для решения частичной проблемы собственных значений и собственных векторов в практических расчетах часто используется метод итераций (степенной метод). На его основе можно определить приближенно собственные значения матрицы [math]A[/math] и спектральный радиус [math]rho(A)= max_{i}bigl|lambda_i(A)bigr|[/math].

Пусть матрица [math]A[/math] имеет [math]n[/math] линейно независимых собственных векторов [math]X^i,~ i=1,ldots,n[/math], и собственные значения матрицы [math]A[/math] таковы, что

[math]rho(A)= bigl|lambda_1(A)bigr|> bigl|lambda_2(A)bigr|geqslant ldotsgeqslant bigl|lambda_n(A)bigr|.[/math]

Алгоритм метода итераций

1. Выбрать произвольное начальное (нулевое) приближение собственного вектора [math]X^{1(0)}[/math] (второй индекс в скобках здесь и ниже указывает номер приближения, а первый индекс без скобок соответствует номеру собственного значения). Положить [math]k=0[/math].

2. Найти [math]X^{1(1)}=AX^{1(0)},~ lambda_1^{(1)}= frac{x_i^{1(1)}}{x_i^{1(0)}}[/math], где [math]i[/math] — любой номер [math]1leqslant ileqslant n[/math], и положить [math]k=1[/math].

3. Вычислить [math]X^{1(k+1)}=Acdot X^{1(k)}[/math].

4. Найти [math]lambda_1^{(k+1)}= frac{x_i^{1(k+1)}}{x_i^{1(k)}}[/math], где [math]x_i^{1(k+1)}, x_i^{1(k)}[/math] — соответствующие координаты векторов [math]X^{1(k+1)}[/math] и [math]X^{1(k)}[/math]. При этом может быть использована любая координата с номером [math]i,~ 1leqslant ileqslant n[/math].

5. Если [math]Delta= bigl|lambda_1^{(k+1)}- lambda_1^{(k)}bigr|leqslant varepsilon[/math], процесс завершить и положить [math]lambda_1cong lambda_1^{k+1}[/math]. Если [math]Delta>varepsilon[/math], положить [math]k=k+1[/math] и перейти к пункту 3.


Замечания

1. Процесс последовательных приближений

[math]begin{aligned}&X^{1(1)}= AX^{1(0)},quad X^{1(2)}= AX^{1(1)}= A^{2}X^{1(0)},quad ldots,\ &X^{1(k)}= AX^{1(k-1)}= AA^{k-1}X^{1(0)}= A^kX^{1(0)},quad ldots end{aligned}[/math]

сходится, т.е. при [math]xtoinfty[/math] вектор [math]X^{1(k)}[/math] стремится к собственному вектору [math]X^1[/math]. Действительно, разложим [math]X^{1(0)}[/math] по всем собственным векторам: [math]textstyle{X^{1(0)}= sumlimits_{i=1}^{n} c_iX^i}[/math]. Так как, согласно (2.4), [math]AX^i= lambda_iX^i[/math], то

[math]begin{aligned}& AX^{1(0)}= X^{1(1)}= sumlimits_{i=1}^{n} c_i lambda_iX^i,quad AX^{1(1)}= A^2X^{1(0)}= X^{1(2)}= sumlimits_{i=1}^{n} c_i lambda_i^2X^i,quad ldots\ &A^kX^{1(0)}= X^{1(k)}= sumlimits_{i=1}^{n} c_i lambda_i^kX^i= lambda_1^k! left[c_1X^1+ c_2{left(frac{lambda_2}{lambda_1}right)!}^k X^2+ ldots+ c_n{left(frac{lambda_n}{lambda_1}right)!}^k X^nright]!. end{aligned}[/math]

При большом [math]k[/math] дроби [math]{left(frac{lambda_2}{lambda_1}right)!}^k, ldots, {left(frac{lambda_n}{lambda_1}right)!}^k[/math] малы и поэтому [math]A^kX^{1(0)}= c_1lambda_1^kX^1[/math], то есть [math]X^{1(k)}to X^1[/math] при [math]ktoinfty[/math]. Одновременно [math]lambda_1= limlimits_{ktoinfty} frac{x_{i}^{1(k+1)}}{x_{i}^{1(k)}}[/math].

2. Вместо применяемой в пункте 4 алгоритма формулы для [math]lambda_1^{(k+1)}[/math] можно взять среднее арифметическое соответствующих отношений для разных координат.

3. Метод может использоваться и в случае, если наибольшее по модулю собственное значение матрицы [math]A[/math] является кратным, т.е.

[math]lambda_1= lambda_2= ldots= lambda_s[/math] и [math]bigl|lambda_1bigr|> bigl|lambda_kbigr|[/math] при [math]k>s[/math].

4. При неудачном выборе начального приближения [math]X^{1(0)}[/math] предел отношения [math]frac{x_i^{1(k+1)}}{x_i^{1(k)}}[/math] может не существовать. В этом случае следует задать другое начальное приближение.

5. Рассмотренный итерационный процесс для [math]lambda_1[/math] сходится линейно, с параметром [math]c=frac{lambda_2}{lambda_1}[/math] и может быть очень медленным. Для его ускорения используется алгоритм Эйткена.

6. Если [math]A=A^T[/math] (матрица [math]A[/math] симметрическая), то сходимость процесса при определении [math]rho(A)[/math] может быть ускорена.

7. Используя [math]lambda_1[/math], можно определить следующее значение [math]lambda_2[/math] по формуле [math]lambda_2= frac{x_i^{1(k+1)}- lambda_1 x_i^{1(k)}}{x_i^{1(k)}- lambda_1 x_i^{1(k-1)}}~ (i=1,2,ldots,n)[/math]. Эта формула дает грубые значения для [math]lambda_2[/math], так как значение [math]lambda_1[/math] является приближенным. Если модули всех собственных значений различны, то на основе последней формулы можно вычислять и остальные [math]lambda_j~(j=3,4,ldots,n)[/math].

8. После проведения нескольких итераций рекомендуется «гасить» растущие компоненты получающегося собственного вектора. Это осуществляется нормировкой вектора, например, по формуле [math]frac{X^{1(k)}}{|X^{1(k)}|_1}[/math].

Пример 2.3. Для матрицы [math]A=begin{pmatrix}5&1&2\ 1&4&1\ 2&1&3 end{pmatrix}[/math] найти спектральный радиус степенным методом с точностью [math]varepsilon=0,,1[/math].

Решение

1. Выбирается начальное приближение собственного вектора [math]X^{(0)}= begin{pmatrix} 1&1&1 end{pmatrix}^T[/math]. Положим [math]k=0[/math].

2. Найдем [math]X^{1(0)}= AX^{1(0)}= begin{pmatrix}5&1&2\ 1&4&1\ 2&1&3end{pmatrix}!cdot! begin{pmatrix}1\1\1end{pmatrix}= begin{pmatrix}8\6\6end{pmatrix}[/math]; [math]lambda_1^{(1)}= frac{x_1^{1(1)}}{x_1^{1(0)}}= frac{8}{1}=8[/math], положим [math]k=1[/math].

3. Вычислим [math]X^{1(2)}= AX^{1(1)}= begin{pmatrix}5&1&2\ 1&4&1\ 2&1&3 end{pmatrix}!cdot! begin{pmatrix}8\6\6end{pmatrix}= begin{pmatrix} 58\38\40 end{pmatrix}[/math].

4. Найдем [math]lambda_1^{(2)}= frac{x_1^{1(2)}}{x_1^{1(1)}}= frac{58}{8}=7,!25[/math].

5. Так как [math]bigl|lambda_1^{(2)}- lambda_1^{(1)}bigr|= 0,!75> varepsilon[/math], то процесс необходимо продолжить. Результаты вычислений удобно представить в виде табл. 10.10.

[math]begin{array}{|c|c|c|c|c|c|}hline phantom{begin{matrix}|\.end{matrix}} k phantom{begin{matrix}|\.end{matrix}} & x_{1}^{1(k)} & x_{2}^{1(k)} & x_{3}^{1(k)} & lambda_1^{(k)} & bigl|lambda_1^{(k)}- lambda_1^{(k-1)}bigr| \ hline 0& 1 & 1 & 1 &- &- \ hline 1& 8& 6& 6 & 8 &- \ hline 2& 58& 38& 40& 7,!25& 0,!75\ hline 3& 408& 250& 274& 7,!034& 0,!116\ hline 4& 2838& 1682& 1888& 6,!9559& 0,!078< varepsilon\ hline end{array}[/math]

Точность по достигнута на четвертой итерации. Таким образом, в качестве приближенного значения [math]lambda_1[/math] берется 6,9559, а в качестве собственного вектора принимается [math]X^1= begin{pmatrix} 2838& 1682& 1888end{pmatrix}^T[/math].

Так как собственный вектор определяется с точностью до постоянного множителя, то [math]X^1[/math] лучше пронормировать, т.е. поделить все его компоненты на величину нормы. Для рассматриваемого примера получим

[math]X^1= frac{1}{2838}cdot! begin{pmatrix}2838\ 1682\ 1888end{pmatrix}= begin{pmatrix}1,!000\ 0,!5927\ 0,!6652 end{pmatrix}!.[/math]

Согласно замечаниям, в качестве собственного значения [math]lambda_1[/math] матрицы можно взять не только отношение

[math]frac{x_1^{1(4)}}{x_1^{1(3)}}= frac{2838}{408}approx 6,!9559[/math], но и [math]frac{x_2^{1(4)}}{x_2^{1(3)}}= frac{1682}{250}approx 6,!7280;~~ frac{x_3^{1(4)}}{x_3^{1(3)}}= frac{1888}{274}approx 6,!8905[/math],

а также их среднее арифметическое [math]frac{6,!9559+6,!728+6,!8905}{3}approx 6,!8581[/math].

Пример 2.4. Найти максимальное по модулю собственное значение матрицы [math]A=begin{pmatrix}2&-1&1\ -1&2&-1\ 0&0&3 end{pmatrix}[/math] и соответствующий собственный вектор.

Решение

1. Зададим начальное приближение [math]X^{1(0)}= begin{pmatrix}1&-1&1 end{pmatrix}^T[/math] и [math]varepsilon=0,!0001[/math].

Выполним расчеты согласно методике (табл. 10.11).

[math]begin{array}{|c|c|c|c|c|c|}hline phantom{begin{matrix}|\.end{matrix}} k phantom{begin{matrix}|\.end{matrix}} & x_{1}^{1(k)} & x_{2}^{1(k)} & x_{3}^{1(k)} & lambda_1^{(k)} & bigl|lambda_1^{(k)}-lambda_1^{(k-1)}bigr| \ hline 0 & 1&-1& 1&-&-\ hline 1 & 4&-4& 1& 4&-\ hline 2 & 13&-13& 1& 3,!25& 0,!75 \ hline 3 & 40&-40& 1& 3,!0769& 0,!17307\ hline 4 & 121& -121& 1& 3,!025& 0,!0519\ hline 5 & 364&-364& 1& 3,!00826& 0,!01673\ hline 6 & 1093&-1093& 1& 3,!002747& 0,!005512\ hline 7 & 3280&-3280 & 1& 3,!000914& 0,!00183\ hline 8 & 9841& -9841& 1& 3,!000304& 0,!000609\ hline 9 & 29524& -29524& 1& 3,!000101& 0,!000202\ hline 10 & 88573& -88573& 1& 3,!000034& 0,!000067\ hline end{array}[/math]

В результате получено собственное значение [math]lambda_1cong 3,!00003[/math] и собственный вектор [math]X^1= begin{pmatrix} 88573&-88573&1end{pmatrix}^T[/math] или после нормировки

[math]X^1= frac{1}{88573}cdot begin{pmatrix} 88573&-88573&1 end{pmatrix}^T = begin{pmatrix} 1&-1&0,!0000113end{pmatrix}^T.[/math]


Метод вращений для нахождения собственных значений

Метод используется для решения полной проблемы собственных значений симметрической матрицы и основан на преобразовании подобия исходной матрицы [math]Ainmathbb{R}^{ntimes n}[/math] с помощью ортогональной матрицы [math]H[/math].

Напомним, что две матрицы [math]A[/math] и [math]A^{(i)}[/math] называются подобными ([math]Asim A^{(i)}[/math] или [math]A^{(i)}sim A[/math]), если [math]A^{(i)}=H^{-1}AH[/math] или [math]A=HA^{(i)}H^{-1}[/math], где [math]H[/math] — невырожденная матрица.

В методе вращений в качестве [math]H[/math] берется ортогональная матрица, такая, что [math]HH^{T}=H^{T}H=E[/math], т.е. [math]H^{T}=H^{-1}[/math]. В силу свойства ортогонального преобразования евклидова норма исходной матрицы [math]A[/math] не меняется. Для преобразованной матрицы [math]A^{(i)}[/math] сохраняется ее след и собственные значения [math]lambda_icolon[/math]

[math]operatorname{tr}A= sum_{i=1}^{n}a_{ii}= sum_{i=1}^{n} lambda_i(A)= operatorname{tr}A^{(i)}.[/math]

При реализации метода вращений преобразование подобия применяется к исходной матрице [math]A[/math] многократно:

[math]A^{(k+1)}= bigl(H^{(k)}bigr)^{-1}cdot A^{(k)}cdot H^{(k)}= bigl(H^{(k)}bigr)^{T}cdot A^{(k)}cdot H^{(k)},quad k=0,1,2,ldots[/math]

(2.6)

Формула (2.6) определяет итерационный процесс, где начальное приближение [math]A^{(0)}=A[/math]. На k-й итерации для некоторого выбираемого при решении задачи недиагонального элемента [math]a_{ij}^{(k)},~ ine j[/math], определяется ортогональная матрица [math]H^{(k)}[/math], приводящая этот элемент [math]a_{ij}^{(k+1)}[/math] (а также и [math]a_{ji}^{(k+1)}[/math]) к нулю. При этом на каждой итерации в качестве [math]a_{ij}^{(k+1)}[/math] выбирается наибольший по модулю. Матрица [math]H^{(k)}[/math] называемая матрицей вращения Якоби, зависит от угла [math]varphi^{(k)}[/math] и имеет вид

Матрица вращения Якоби

В данной ортогональной матрице элементы на главной диагонали единичные, кроме [math]h_{ii}^{(k)}= cosvarphi^{(k)}[/math] и [math]h_{jj}^{(k)}=cosvarphi^{(k)}[/math], а остальные элементы нулевые, за исключением [math]h_{ij}^{(k)}=-sinvarphi^{(k)}[/math], [math]h_{ji}^{(k)}=sinvarphi^{(k)}[/math] ([math]h_{ij}[/math] -элементы матрицы [math]H[/math]).

Угол поворота [math]varphi^{(k)}[/math] определяется по формуле

[math]operatorname{tg}2varphi^{(k)}= frac{2a_{ij}^{(k)}}{a_{ii}^{(k)}-a_{jj}^{(k)}}= overline{P}_k,;qquad varphi^{(k)}= frac{1}{2}operatorname{arctg}overline{P}_k,,[/math]

(2.7)

где [math]|2varphi^{(k)}|leqslant frac{pi}{2},~ i<j[/math] ([math]a_{ij}[/math] выбирается в верхней треугольной наддиагональной части матрицы [math]A[/math]).

В процессе итераций сумма квадратов всех недиагональных элементов [math]sigms(A^{(k)})[/math] при возрастании [math]k[/math] уменьшается, так что [math]sigms(A^{(k+1)})< sigma(A^{(k)})[/math]. Однако элементы [math]a_{ij}^{(k)}[/math] приведенные к нулю на k-й итерации, на последующей итерации немного возрастают. При [math]ktoinfty[/math] получается монотонно убывающая ограниченная снизу нулем последовательность [math]sigma(A^{(1)})> sigma(A^{(2)})> ldots> sigma(A^{(k)})>ldots[/math]. Поэтому [math]sigma(A^{(k)})to0[/math] при [math]ktoinfty[/math]. Это и означает сходимость метода. При этом [math]A^{(k)}to Lambda= operatorname{diag}(lambda_1,ldots,lambda_n)[/math].

Замечание. В двумерном пространстве с введенной в нем системой координат [math]Oxy[/math] с ортонормированным базисом [math]{vec{i},vec{j}}[/math] матрица вращения легко получается из рис. 2.1, где система координат [math]Ox’y'[/math] повернута на угол [math]varphicolon[/math]

[math]begin{cases}vec{i},’= vec{i}cos varphi+ vec{j}sin varphi,\[2pt] vec{j},’=-vec{i}sin varphi+ vec{j}cos varphi. end{cases}[/math]

Таким образом, для компонент [math]vec{i},’,, vec{j},'[/math] будем иметь [math]bigl(vec{i},’,vec{j},’bigr)= bigl(vec{i},vec{j}bigr)cdot! begin{pmatrix} cos varphi&-sin varphi\ sin varphi& cos varphiend{pmatrix}[/math]. Отсюда следует, что в двумерном пространстве матрица вращения имеет вид [math]H= begin{pmatrix} cos varphi&-sin varphi\ sin varphi& cos varphiend{pmatrix}[/math]. Отметим, что при [math]n=2[/math] для решения задачи требуется одна итерация.

Графическая интерпретация матрицы вращения Якоби


Алгоритм метода вращений

1. Положить [math]k=0,~ A^{(0)}=A[/math] и задать [math]varepsilon>0[/math].

2. Выделить в верхней треугольной наддиагональной части матрицы [math]A^{(k)}[/math] максимальный по модулю элемент [math]a_{ij}^{(k)},~ i<j[/math].

Если [math]|a_{ij}^{(k)}|leqslant varepsilon[/math] для всех [math]ine j[/math], процесс завершить. Собственные значения определяются по формуле [math]lambda_i(A^{(k)})=a_{ii}^{(k)},~ i=1,ldots,n[/math].

Собственные векторы [math]X^i[/math] находятся как i-e столбцы матрицы, получающейся в результате перемножения:

[math]nu_k= H^{(0)}cdot H^{(1)}cdot H^{(2)}cdot ldotscdot H^{(k-1)}= bigl( X^1,X^2,X^3,ldots,X^nbigr).[/math]

Если [math]bigl|a_{ij}^{(k)}bigr|>varepsilon[/math], процесс продолжается.

3. Найти угол поворота по формуле [math]varphi^{(k)}= frac{1}{2} operatorname{arctg} frac{2a_{ij}^{(k)}}{a_{ii}^{(k)}-a_{jj}^{(k)}}[/math].

4. Составить матрицу вращения [math]H^{(k)}[/math].

5. Вычислить очередное приближение [math]A^{(k+1)}= bigl(H^{(k)}bigr)^T A^{(k)} H^{(k)}[/math].Положить [math]k=k+1[/math] и перейти к пункту 2.

Замечания

1. Используя обозначение [math]overline{P}_k= frac{2a_{ij}^{(k)}}{a_{ii}^{(k)}-a_{jj}^{(k)}}[/math], можно в пункте 3 алгоритма вычислять элементы матрицы вращения по формулам

[math]sinvarphi^{(k)}= operatorname{sign}overline{P}_kcdot sqrt{frac{1}{2}!left(1-frac{1}{sqrt{1+overline{P}_{k}^{;2}}}right)},,qquad cosvarphi^{(k)}= sqrt{frac{1}{2}!left(1+ frac{1}{sqrt{1+overline{P}_{k}^{;2}}}right)},.[/math]

2. Контроль правильности выполнения действий по каждому повороту осуществляется путем проверки сохранения следа преобразуемой матрицы.

3. При [math]n=2[/math] для решения задачи требуется одна итерация.

Пример 2.5. Для матрицы [math]A=begin{pmatrix} 2&1\1&3 end{pmatrix}[/math] методом вращений найти собственные значения и собственные векторы.

Решение

1. Положим [math]k=0,~A^{(0)}=A= begin{pmatrix} 2&1\1&3 end{pmatrix}!,~ varepsilon=10^{-10}[/math].

2°. Выше главной диагонали имеется только один элемент [math]a_{ij}=a_{12}=1[/math].

3°. Находим угол поворота матрицы по формуле (2.7), используя в расчетах 11 цифр после запятой в соответствии с заданной точностью:

[math]operatorname{tg}2varphi^{(0)}= frac{2a_{ij}}{a_{ii}-a_{jj}}= frac{2}{2-3}=-2quad Rightarrowquad! left[begin{aligned} sinvarphi^{(0)}&approx -0,!52573111212,;\ cosvarphi^{(0)}&approx 0,!85065080835,.end{aligned}right.[/math]

4°. Сформируем матрицу вращения:

[math]H^{(0)}= begin{pmatrix}cosvarphi^{(0)}& -sinvarphi^{(0)}\[2pt] sinvarphi^{(0)}& cosvarphi^{(0)} end{pmatrix}= begin{pmatrix} 0,!85065080835 & 0,!52573111212 \[2pt] -0,!52573111212 & 0,!85065080835 end{pmatrix}!.[/math]

5°. Выполним первую итерацию:

[math]begin{aligned}A^{(1)}= bigl(H^{(0)}bigr)^Tcdot A^{(0)}cdot H^{(0)}&= begin{pmatrix}0,!85065080835 & -0,!52573111212 \[2pt] 0,!52573111212 & 0,!85065080835 end{pmatrix}!cdot A^{(0)}cdot begin{pmatrix}0,!85065080835 & 0,!52573111212 \[2pt] -0,!52573111212 & 0,!85065080835end{pmatrix}=\ &= begin{pmatrix} 1,!38196601125& -4,!04620781325cdot10^{-12}\ -4,!04587474634cdot10^{-12}& 3,!61803398874 end{pmatrix}!. end{aligned}[/math]

Очевидно, след матрицы с заданной точностью сохраняется, т.е. [math]sum_{i=1}^{2}a_{ii}^{(0)}= sum_{i=1}^{2}a_{ii}^{(0)}=5[/math]. Положим [math]k=1[/math] и перейдем к пункту 2.

2. Максимальный по модулю наддиагональный элемент [math]|a_{12}|= 4,!04620781325cdot10^{-12} < varepsilon=10^{-10}[/math]. Для решения задачи (подчеркнем, что [math]n=2[/math]) с принятой точностью потребовалась одна итерация, полученную матрицу можно считать диагональной. Найдены следующие собственные значения и собственные векторы:

[math]lambda_1= 1,!38196601125,.quad lambda_2=3,!61803398874,; qquad X^1= begin{pmatrix} 0,!85065080835\ -0,!52573111212end{pmatrix}!;quad X^2=begin{pmatrix} 0,!52573111212\ 0,!85065080835 end{pmatrix}!.[/math]

Пример 2.6. Найти собственные значения и собственные векторы матрицы [math]A=begin{pmatrix}5&1&2\ 1&4&1\ 2&1&3 end{pmatrix}[/math].

Решение

1. Положим [math]k=0,~ A^{(0)}=A,~ varepsilon= 0,!001[/math].

2°. Выделим максимальный по модулю элемент в наддиагональнои части: [math]a_{13}^{(0)}=2[/math]. Так как [math]a_{13}=2> varepsilon=0,!001[/math], то процесс продолжается.

3°. Находим угол поворота:

[math]varphi^{(0)}= frac{1}{2}operatorname{arctg} frac{2a_{13}^{(0)}}{a_{11}^{(0)}-a_{33}^{(0)}}= frac{1}{2}operatorname{arctg}frac{4}{5-3}= frac{1}{2} operatorname{arctg}2approx 0,!553574quad Rightarrowquad! left[begin{aligned} sinvarphi^{(0)}&approx 0,!52573,;\ cosvarphi^{(0)}&approx 0,!85065,. end{aligned}right.[/math]

4°. Сформируем матрицу вращения: [math]H^{(0)}= begin{pmatrix}0,!85065&0&-0,!52573\ 0&1&0\ 0,!52573&0&0,!85065 end{pmatrix}[/math].

5°. Выполним первую итерацию: [math]A^{(1)}= bigl(H^{(0)}bigr)^T A^{(0)}H^{(0)}= begin{pmatrix} 6,!236&1,!376&2,!33cdot10^{-6}\ 1,!376&4&0,!325\ 2,!33cdot10^{-6}&0,!325&1,!764 end{pmatrix}[/math]. Положим [math]k=1[/math] и перейдем к пункту 2.

2(1). Максимальный по модулю наддиагональный элемент [math]a_{12}^{(1)}=1,!376[/math]. Так как [math]a_{12}^{(1)}> varepsilon=0,!001[/math], процесс продолжается.

3(1). Найдем угол поворота:

[math]varphi^{(1)}= frac{1}{2} operatorname{arctg} frac{2a_{12}^{(1)}}{a_{11}^{(1)}-a_{22}^{(1)}}= frac{1}{2} operatorname{arctg}frac{2cdot1,!376}{6,!236-4}= frac{1}{2} operatorname{arctg}1,!230769approx 0,!444239quad Rightarrowquad! left[begin{aligned} sinvarphi^{(1)}approx 0,!429770,;\ cosvarphi^{(1)}approx 0,!902937,. end{aligned}right.[/math]

4(1). Сформируем матрицу вращения: [math]H^{(1)}= begin{pmatrix} 0,!902937&-0,!429770&0\ 0,!429770&0,!902937&0\ 0&0&1 end{pmatrix}[/math].

5(1). Выполним вторую итерацию: [math]A^{(2)}= bigl(H^{(1)}bigr)^T A^{(1)}H^{(1)}= begin{pmatrix} 6,!891& 2,!238cdot10^{-4}&0,!14\ 2,!238cdot10^{-4}& 3,!345&0,!293\ 0,!14&0,!293&1,!764 end{pmatrix}[/math]. Положим [math]k=2[/math] и перейдем к пункту 2.

2(2). Максимальный по модулю наддиагональный элемент [math]a_{23}^{(2)}=0,!293> varepsilon=0,!001[/math].

3(2). Найдем угол поворота:

[math]varphi^{(2)}= frac{1}{2} operatorname{arctg} frac{2a_{23}^{(2)}}{a_{22}^{(2)}-a_{33}^{(2)}}= frac{1}{2} operatorname{arctg}frac{2cdot0,!293}{3,!345-1,!764}= frac{1}{2} operatorname{arctg}0,!370651approx 0,!177476quad Rightarrowquad! left[begin{aligned} sinvarphi^{(2)}approx 0,!1765460,;\ cosvarphi^{(2)}approx 0,!9842924,. end{aligned}right.[/math]

4(2). Сформируем матрицу вращения [math]H^{(2)}= begin{pmatrix}1&0&0\ 0&0,!9842924& -0,!1765460\ 0& 0,!1765460& 0,!9842924end{pmatrix}[/math].

5(2). Выполним третью итерацию и положим [math]k=3[/math] и перейдем к пункту 2:

[math]A^{(3)}=bigl(H^{(2)}bigr)^Tcdot A^{(2)}cdot H^{(2)}= ldots= begin{pmatrix} 6,!891&0,!025&0,!138\ 0,!025&3,!398&3,!375cdot 10^{-7}\ 0,!138&3,!375cdot10^{-7}& 1,!711 end{pmatrix}!.[/math]

2(3). Максимальный по модулю наддиагональный элемент [math]a_{13}^{(3)}= 0,!138>varepsilon[/math].

3(3). Найдем угол поворота:

[math]varphi^{(3)}= frac{1}{2}operatorname{arctg} frac{2cdot a_{13}^{(3)}}{a_{11}^{(3)}-a_{33}^{(3)}}= frac{1}{2}operatorname{arctg} frac{2cdot0,!138}{6,!891-1,!711}= frac{1}{2}operatorname{arctg}0,!05328approx 0,!026615 quadRightarrowquad! left[begin{aligned} sinvarphi^{(3)}approx 0,!026611,;\ cosvarphi^{(3)}approx 0,!999646,. end{aligned}right.[/math]

4(3). Сформируем матрицу вращения: [math]H^{(3)}= begin{pmatrix} 0,!999646&0&-0,!026611\ 0&1&0\ 0,!026611&0&0,!999646 end{pmatrix}[/math].

5(3). Выполним четвертую итерацию и положим [math]k=4[/math] и перейдем к пункту 2:

[math]A^{(4)}=bigl(H^{(3)}bigr)^Tcdot A^{(3)}cdot H^{(3)}= ldots= begin{pmatrix} 6,!895&0,!025&8,!406cdot10^{-6}\ 0,!025&3,!398&-6,!649cdot10^{-6}\ 8,!406cdot10^{-6}& -6,!649cdot10^{-6}&1,!707 end{pmatrix}!.[/math]

2(4). Так как [math]a_{12}^{(4)}=0,!025>varepsilon[/math], процесс повторяется

3(4). Найдем угол поворота

[math]varphi^{(4)}= frac{1}{2}operatorname{arctg} frac{2cdot a_{12}^{(4)}}{a_{11}^{(4)}-a_{22}^{(4)}}= frac{1}{2}operatorname{arctg} frac{2cdot0,!025}{6,!895-3,!398}approx 0,!0071484 quadRightarrowquad! left[begin{aligned} sinvarphi^{(4)}approx 0,!0071483,;\ cosvarphi^{(4)}approx 0,!9999744,. end{aligned}right.[/math]

4(4). Сформируем матрицу вращения: [math]H^{(4)}= begin{pmatrix} 0,!9999744&-0,!0071483&0\ 0,!0071483&0,!9999744&0\ 0&0&1 end{pmatrix}[/math].

5(4). Выполним пятую итерацию и положим [math]k=5[/math] и перейдем к пункту 2:

[math]A^{(5)}=bigl(H^{(4)}bigr)^Tcdot A^{(4)}cdot H^{(4)}= ldots= begin{pmatrix} 6,!895&4,!774cdot10^{-7}&3,!653cdot10^{-6}\ 4,!774cdot10^{-7}&3,!398&-6,!649cdot10^{-4}\ 3,!653cdot10^{-6}& -6,!649cdot10^{-4}&1,!707 end{pmatrix}!.[/math]

2(5). Так как наибольший по модулю наддиагональный элемент удовлетворяет условию [math]bigl|-6,!649cdot10^{-4}bigr|<varepsilon=0,!001[/math], процесс завершается.

Собственные значения: [math]lambda_1cong a_{11}^{(5)}= 6,!895,,~ lambda_2cong a_{22}^{(5)}=3,!398,,~ lambda_3cong a_{33}^{(5)}=1,!707,,[/math]. Для нахождения собственных векторов вычислим

[math]nu_5=H^{(0)}cdot H^{(1)}cdot H^{(2)}cdot H^{(3)}cdot H^{(4)}= begin{pmatrix} 0,!753&-0,!458&-0,!473\ 0,!432&0,!886&-0,!171\ 0,!497&-0,!076&0,!864 end{pmatrix}!,[/math]

отсюда [math]X^1=begin{pmatrix} 0,!753\0,!432\0,!497 end{pmatrix}!,~ X^2=begin{pmatrix}-0,!458\ 0,!886\ -0,!076 end{pmatrix}!,~ X^3=begin{pmatrix}-0,!473\-0,!171\0,!864 end{pmatrix}[/math] или после нормировки

[math]X^1=begin{pmatrix}1\0,!5737\0,!660end{pmatrix}!,qquad X^2=begin{pmatrix}-0,!517\ 1\ -0,!0858end{pmatrix}!,qquad X^3=begin{pmatrix}-0,!5474\-0,!1979\1 end{pmatrix}!.[/math]

Математический форум (помощь с решением задач, обсуждение вопросов по математике).

Кнопка "Поделиться"

Если заметили ошибку, опечатку или есть предложения, напишите в комментариях.

Содержание

Характеристический полином, собственные числа, собственные векторы матрицы

§

В настоящем разделе $ n_{} $ означает порядок квадратной матрицы $ A_{} $.

Характеристический полином

определяется для произвольной квадратной матрицы $ A_{} $ как1)
$ det (A_{}-lambda E) $, где $ E_{} $ – единичная матрица одинакового с $ A_{} $ порядка.

П

Пример. Для $ n=2_{} $:

$$ det (A-lambda E)=
begin{vmatrix}
a_{11}-lambda & a_{12}\
a_{21}& a_{22}-lambda
end{vmatrix}=
$$
$$
=lambda^2-(a_{11}+a_{22})lambda + (a_{11}a_{22}-a_{12}a_{21}) ;
$$
для $ n=3_{} $:
$$
det (A-lambda E)=
begin{vmatrix}
a_{11}-lambda & a_{12} & a_{13}\
a_{21}& a_{22}-lambda & a_{23} \
a_{31}& a_{32} & a_{33}-lambda
end{vmatrix}=
$$
$$
=-lambda^3+(a_{11}+a_{22}+a_{33})lambda^2 —
$$
$$
-left {
begin{vmatrix}
a_{11}& a_{12}\
a_{21}& a_{22}
end{vmatrix}
+begin{vmatrix}
a_{22}& a_{23}\
a_{32}& a_{33}
end{vmatrix}+
begin{vmatrix}
a_{11}& a_{13}\
a_{31}& a_{33}
end{vmatrix}
right }lambda+
det A .
$$

Т

Теорема 1.

$$ det (A-lambda E)=
(-1)^nBigg(
lambda^n — lambda^{n-1}sum_{1le jle n} a_{jj}
+ lambda^{n-2}sum_{1le j_1< j_2 le n} left|begin{array}{ll}
a_{j_1j_1}& a_{j_1j_2}\
a_{j_2j_1}& a_{j_2j_2}
end{array}right| — dots +
$$
$$
+(-1)^k lambda^{n-k}
sum_{1le j_1< j_2 < dots < j_kle n} left|begin{array}{llll}
a_{j_1j_1}& a_{j_1j_2} & dots & a_{j_1j_k}\
a_{j_2j_1}& a_{j_2j_2} & dots & a_{j_2j_k}\
vdots & & & vdots \
a_{j_kj_1}& a_{j_kj_2} & dots & a_{j_kj_k}
end{array}right|+ dots + (-1)^n det A
Bigg) .
$$
Образно говоря, коэффициент при $ (-1)^{k}lambda^{n-k} $ получается
суммированием всех миноров $ k_{} $-го порядка матрицы $ A_{} $, построенных на
элементах, стоящих в строках и столбцах с одинаковыми номерами..

Такой минор матрицы
$$
left|begin{array}{cccc}
a_{j_1j_1} & a_{j_1j_2} & dots & a_{j_1j_k} \
a_{j_2j_1} & a_{j_2j_2} & dots & a_{j_2j_k} \
vdots & & ddots & vdots \
a_{j_kj_1} & a_{j_kj_2} & dots & a_{j_kj_k}
end{array}
right|
, quad 1le j_1<j_2< dots < j_k le n
$$
в настоящем ресурсе называется ведущим минором ($k$-го порядка). См. замечание о терминологии



ЗДЕСЬ.

Результат теоремы имеет исключительно теоретическое значение: практическое вычисление характеристического полинома матрицы большого порядка по этой теореме обычно крайне неэффективно. Методы практического вычисления характеристического полинома разбираются



НИЖЕ.

П

Пример. Характеристический полином матрицы Фробениуса

$$
mathfrak F=
left( begin{array}{lllllll}
0 & 1 & 0 & 0 & dots & 0 & 0 \
0 & 0 & 1 & 0 & dots & 0 & 0 \
0 & 0 & 0 & 1 & dots & 0 & 0 \
vdots& &&&ddots & & vdots \
0 & 0 & 0 & 0 & dots & 0 & 1 \
a_n & a_{n-1} & a_{n-2} & & dots & a_2 & a_1
end{array} right)_{n times n}
$$
равен $ (-1)^n(lambda^n-a_1lambda^{n-1}-dots-a_{n}) $.

Характеристический полином линейного оператора

определяется как характеристический полином матрицы этого оператора в произвольном базисе линейного пространства, в котором этот оператор задан. Подробнее



ЗДЕСЬ.

Характеристический полином линейного однородного разностного уравнения

$ n_{} $-го порядка
$$
x_{n+K}=a_1 x_{n+K-1}+ dots+ a_n x_K, quad a_n ne 0,
$$
определяется как
$$ lambda^n — a_1 lambda^{n-1} — dots — a_n . $$
Подробнее



ЗДЕСЬ.

Свойства

Т

Теорема 2. Характеристический полином матрицы не меняется


1.

при ее транспонировании:
$$ det (A-lambda E) = det (A^{top}-lambda E_{}) , ;$$


2.

при переходе к подобной матрице: если $ B=C^{-1}AC^{} $ при произвольной неособенной матрице $ C_{} $, то
$$ det (A-lambda E) equiv det (B-lambda E_{}) , . $$

Т

Теорема 3. Пусть матрица $ A_{} $ имеет порядок $ mtimes n_{} $, а $ B_{} $ — порядок $ ntimes m_{} $. Тогда эти матрицы допускают умножение в любом порядке, т.е. определены $ AB_{} $ и $ BA_{} $ и оба произведения будут квадратными матрицами — порядков $ m_{} $ и $ n_{} $ соответственно. Тогда характеристические полиномы этих произведений различаются лишь на степень $ lambda_{} $:

$$
lambda^n det (AB — lambda E_{m times m})equiv lambda^m det (BA — lambda E_{n times n}) .
$$

=>

Если матрицы $ A_{} $ и $ B_{} $ — квадратные одинакового порядка, то характеристические полиномы матриц $ AB_{} $ и $ BA_{} $ тождественны.

Т

Теорема 4. Если характеристический полином матрицы $ A_{} $ равен

$$ f(lambda)=(-1)^n lambda^n+a_1lambda^{n-1}+dots+a_{n-1}lambda+a_n $$
и $ a_{n} ne 0 $, то характеристический полином матрицы $ A^{-1}_{} $ равен
$$ f^{ast}(lambda)=frac{(-lambda)^n}{a_n} f(1/lambda) = frac{(-1)^n}{a_n} left[ (-1)^n+a_1 lambda + dots+
a_{n-1}lambda^{n-1}+a_nlambda^{n} right] . $$

Теорема Гамильтона-Кэли

Т

Теорема 5. Результатом подстановки в характеристический полином $ det (A_{}-lambda E) $ самой матрицы $ A_{} $ будет нулевая матрица:

$$
det (A-lambda E)= (-1)^n lambda^n +a_1 lambda^{n-1}+dots+a_{n-1}lambda+ a_n Rightarrow
$$
$$
Rightarrow
(-1)^n A^n +a_1 A^{n-1}+dots+a_{n-1}A+ a_n E = {mathbb O}_{ntimes n} .
$$

В литературе эта теорема обычно приводится в иной формулировке:

матрица является корнем своего характеристического полинома.

Доказательство



ЗДЕСЬ.

П

Пример. Для $ n_{}=2 $:

$$
left(begin{array}{ll} a_{11} & a_{12} \ a_{21} & a_{22}
end{array} right)^2 — (a_{11}+a_{22})left(begin{array}{ll} a_{11} & a_{12} \ a_{21} & a_{22}
end{array} right) +
(a_{11}a_{22}-a_{12}a_{21})
left(begin{array}{ll} 1 & 0 \ 0 & 1
end{array} right) = left(begin{array}{ll} 0 & 0 \ 0 & 0
end{array} right) .
$$

Собственное число

Собственное (или характеристическое) число2) определяется для квадратной матрицы $ A_{} $ как произвольный корень ее характеристического полинома $ det (A_{}-lambda E) $. Уравнение
$$
det(A-lambda E)=0
$$
называется характеристическим уравнением матрицы $ A $.

И

Понятие характеристического уравнения было введено Коши в 1840 г. В литературе XIX века известно также как вековое уравнение.

Набор всех собственных чисел матрицы $ A_{} $ (с учетом их кратностей) называется спектром матрицы3) (таким образом спектр матрицы $ A_{} $ порядка $ n_{} $ всегда состоит из $ n_{} $ чисел, часть из которых могут быть одинаковыми). Максимальный из модулей собственных чисел матрицы $ A_{} $ называется ее спектральным радиусом4), он иногда обозначается $ rho(A) $.

П

Пример. Найти спектр матрицы

$$
A=
left(begin{array}{rrrr}
0&1&2&3\
-1&0&4&7\
-2&-4&0&2\
-3&-7&-2&0
end{array}right).
$$
Решение. Характеристический полином
$$ det (A-lambda E)=left|begin{array}{rrrr}
-lambda&1&2&3\
-1&-lambda&4&7\
-2&-4&-lambda&2\
-3&-7&-2&-lambda
end{array}right|=lambda^4+
83lambda^2
$$
имеет корни $ lambda_1=0, lambda_2 = {mathbf i}sqrt{83}, lambda_3 = — {mathbf i} sqrt{83} $, причем $ lambda_{1} $ — второй кратности.

Ответ. Спектр матрицы $ A_{} $:
$ {0,0, {mathbf i} sqrt{83},- {mathbf i} sqrt {83} } $. Спектральный радиус матрицы $ A_{} $: $ rho(A)= sqrt {83} $.

Т

Теорема 6. Если $ {lambda_{1},lambda_{2},dots,lambda_{n} } $ — спектр матрицы $ A_{} $, то

$$ lambda_1+lambda_{2}+dots+lambda_n = operatorname{Sp}(A)=a_{11}+a_{22}+dots+a_{nn}, $$
$$ lambda_1cdotlambda_{2}times dots times lambda_n = (-1)^ndet (A) . $$

Доказательство следует из представления характеристического полинома через миноры матрицы и формул Виета.


Можно, разумеется, привести еще $ n-2 $ зависимостей между собственными числами матрицы и ее минорами, но они редко нужны — а вот равенства из теоремы полезно запомнить.

=>

Для того, чтобы матрица $ A_{} $ была неособенной необходимо и достаточно, чтобы среди ее собственных чисел не было нулевого.

Т

Теорема 7. Пусть $ g(x)=b_{0}x^m+dots+b_m in {mathbb C}[x] $ — произвольный полином. Вычислим полином от матрицы $ A_{} $:

$$ g(A)=b_{0}A^m+dots+b_m E , . $$
Тогда если $ {lambda_{1},dots,lambda_{n} } $ — спектр матрицы $ A_{} $, то $ {g(lambda_{1}),dots,g(lambda_n) } $ — спектр матрицы $ g(A_{}) $.

=>

Результат теоремы обобщается и на более широкий класс функций $ g_{}(x) $ — фактически на любую функцию, которая, вместе со своими производными, может быть определена на спектре матрицы $ A_{} $. В частности, если $ det A_{} ne 0 $, то спектр матрицы $ A^{-1}_{} $ совпадает с $ {1/lambda_j}_{j=1}^n $.

=>

Имеет место следующее равенство, связывающее степени матрицы $ A_{} $ с суммами Ньютона ее характеристического полинома:

$$ operatorname{Sp}(A^k)=lambda_1^k+dots+lambda_n^k . $$
Здесь $ operatorname{Sp}_{} $ обозначает след матрицы (т.е. сумму ее диагональных элементов). Утверждение остается справедливым и для отрицательных показателей $ k_{} $ при условии, что $ det A_{} ne 0 $.

=>

Имеет место следующее равенство:

$$ det g(A) = (-1)^{mn} {mathcal R}(f,g_{}) , $$
где $ {mathcal R}(f,g_{}) $ означает результант полиномов $ f(x) =det (A-x_{} E) $ и $ g_{}(x) $.

Т

Теорема 8. Собственные числа вещественной симметричной матрицы $ A_{} $ все вещественны.

Доказательство



ЗДЕСЬ.

Т

Теорема 9. Собственные числа вещественной кососимметричной матрицы $ A_{} $ все мнимы, за исключением, возможно, $ lambda_{} = 0 $.

Доказательство



ЗДЕСЬ.

Т

Теорема 10. Собственные числа вещественной ортогональной матрицы все равны $ 1_{} $ по абсолютной величине (модулю). Характеристический полином ортогональной матрицы является возвратным если $ +1 $ не является его корнем или является корнем четной кратности. Хотя бы одно собственное число ортогональной матрицы нечетного порядка равно $ +1 $ или $ (-1) $.

Доказательство



ЗДЕСЬ.

Т

Теорема 11. Спектр циклической матрицы

$$
left(begin{array}{lllll}
a_1 & a_2 & a_3 & dots & a_n \
a_n & a_1 & a_2 & dots & a_{n-1} \
a_{n-1} & a_n & a_1 & dots & a_{n-2} \
vdots & & & & vdots \
a_2 & a_3 & a_4 & dots & a_1
end{array}
right) .
$$
совпадает с набором чисел
$$ {f(1),f(varepsilon_1), dots, f(varepsilon_{n-1}) } ,$$
при
$$ f(x)=a_{1}+a_2x+a_3x^2+dots+a_nx^{n-1} $$
и
$$
varepsilon_k=cos frac{2,pi k}{n} + {mathbf i} sin frac{2,pi k}{n}
$$
корне n-й степени из единицы.

Доказательство



ЗДЕСЬ.

Локализация собственных чисел

Т

Теорема 12. [1]. Собственные числа матрицы являются непрерывными функциями ее элементов. Иначе: пусть

$$A=left[a_{jk} right]_{j,k=1}^n quad , quad B=left[b_{jk} right]_{j,k=1}^n
.
$$
Обозначим
$$M= max_{j,kin{1,dots,n}} left{|a_{jk} |, |b_{jk} | right} quad ,
quad delta = frac{1}{nM}sum_{j,k=1}^n |a_{jk} — b_{jk} | . $$
Тогда любому собственному числу $ lambda_{ast}^{} $ матрицы $ A_{} $ можно поставить
в соответствие такое собственное число
$ mu_{ast}^{} $ матрицы $ B_{} $, что
$$ |lambda_{ast}-mu_{ast} | le (n+2) M sqrt[n]{delta} . $$

Собственно факт непрерывной зависимости собственных чисел от элементов матрицы следует из представления характеристического полинома из теоремы



ПУНКТА — коэффициенты этого полинома полиномиально (и, следовательно, непрерывно) зависят от элементов матрицы.
Далее используем теорему о непрерывной зависимости корней полинома от его коэффициентов.

Выясним теперь на примере, насколько малым может быть возмущение элементов матрицы чтобы сохранились хотя бы количество вещественных корней ее характеристического полинома.

П

Пример [Уилкинсон] [2]. Найти собственные числа матрицы

$$
A=
left(
begin{array}{cccccc}
20 & 20 & & & & \
& 19 & 20 & & & \
& & 18 & 20 & & \
& & & ddots & ddots & \
& & & & 2 & 20 \
{color{Red} varepsilon } & & & & & 1 \
end{array}
right)_{20times 20}
$$
при $ {color{Red} varepsilon }=10^{-10} $
(все неуказанные элементы матрицы считаются равными нулю).

Решение. Характеристический полином
$$
det(A-lambda E) = prod_{j=1}^{20} (j-lambda) — 20^{19} {color{Red} varepsilon } =
$$
$$
=lambda^{20}-{scriptstyle 210},lambda^{19}+{scriptstyle 20615},lambda^{18}-{scriptstyle 1256850}, lambda^{17}
+{scriptstyle 53327946}, lambda^{16}-{scriptstyle 1672280820}, lambda^{15}+
{scriptstyle 40171771630}, lambda^{14}-{scriptstyle 756111184500}, lambda^{13}+
$$
$$
+{scriptstyle 11310276995381}, lambda^{12} —
{scriptstyle 135585182899530}, lambda^{11}
+{scriptstyle 1307535010540395}, lambda^{10}-{scriptstyle 10142299865511450}, lambda^9 +
$$
$$
+{scriptstyle 63030812099294896}, lambda^8 —
{scriptstyle 311333643161390640}, lambda^7+{scriptstyle 1206647803780373360}, lambda^6 -{scriptstyle 3599979517947607200}, lambda^5
+{scriptstyle 8037811822645051776}, lambda^4-
$$
$$
-{scriptstyle 12870931245150988800}, lambda^3
+{scriptstyle 13803759753640704000}, lambda^2
-{scriptstyle 8752948036761600000},lambda +{scriptstyle 2432377720176640000}
$$
очень похож на полином из другого



ПРИМЕРА Уилкинсона. Он имеет корни
$$
lambda_1=0.995754, lambda_2=2.109241, lambda_3=2.574881,
$$
$$
lambda_{4,5}=3.965331pm 1.087735, mathbf i, lambda_{6,7}=5.893977pm 1.948530 , mathbf i,
$$
$$
lambda_{8,9}=8.118073 pm
2.529182 , mathbf i,
lambda_{10,11}=10.5pm 2.733397 , mathbf i,
$$
$$
lambda_{12,13}=12.881926pm 2.529182 , mathbf i, lambda_{14,15}=15.106022 pm 1.948530
, mathbf i, $$
$$
lambda_{16,17}=17.034669pm 1.087735 , mathbf i,
$$
$$
lambda_{18}=18.425118, lambda_{19}=18.890758, lambda_{20}=20.004245 .
$$
Итак, нановозмущение5) в одном-единственном элементе матрицы приводит к существенному изменению спектра: из $ 20 $ вещественных собственных чисел «остаются в живых» только $ 6_{} $; кроме того, у образовавшихся мнимых корней оказываются достаточно большими мнимые части. В данном примере допустимые возмущения для $ {color{Red} varepsilon } $, т.е. такие, при
которых сохранится
свойство вещественности всех корней характеристического полинома, находятся в пределах6)
$$ -8.636174times 10^{-14} < {color{Red} varepsilon } le frac{685872258640569}{8796093022208000000000000000} approx +7.797464 times 10^{-14} .$$



Т

Теорема 13 [Гершгорин].7) Обозначим $ mathbb D_{j} $ круг на комплексной
плоскости
$ mathbb C_{} $ с центром в точке $ a_{jj}^{} $ и радиуса

$$ r_j=sum_{ell=1 atop ellne j}^n left|a_{j ell}right| .$$
Тогда спектр матрицы $ A_{} $ лежит внутри объединения этих кругов:
$$ {lambda_1,dots, lambda_n } subset bigcup_{j=1}^n mathbb D_j . $$
Иными словами: любое собственное число матрицы должно удовлетворять хотя бы одному из
неравенств

$$ |z- a_{jj} | < r_j . $$

Доказательство



ЗДЕСЬ

П

Пример. Построить круги Гершгорина для матрицы

$$ A=left(
begin{array}{crr}
-1+3,{mathbf i} & 2- {mathbf i} & 3+2, {mathbf i} \
-1+{mathbf i} & 4+ {mathbf i} & 3, {mathbf i} \
-1& 2-2,{mathbf i}& -2-3, {mathbf i}
end{array}
right) . $$

Решение.
$$|lambda + 1 — 3,{mathbf i} |le | 2-{mathbf i} |+| 3+2,{mathbf i} |=sqrt{5}+sqrt{13}, $$
$$|lambda — 4 — {mathbf i} |le 3+sqrt{2}, $$
$$ |lambda + 2+ 3, {mathbf i} |le 1 + 2sqrt{2} . $$

Проверка. Собственные числа матрицы $ A_{} $ (на рисунке обозначены красными крестиками):
$$ { -2.509081750-3.442241533,{mathbf i} , -1.041999986+2.655757676,{mathbf i} , 4.551081736+1.786483857, {mathbf i} } .$$

Локализация вещественных собственных чисел

Симметричная матрица

Т

Теорема 14 [Коши].
Для вещественной симметричной матрицы $ A_{} $ число ее собственных чисел, лежащих на интервале $ ]a,b_{}[ $, определяется по формуле:

$$operatorname{nrr} { det (A-lambda E) =0 | a< lambda<b }= $$
$$= {mathcal P}(1, H_1(a), H_2(a),dots, H_n(a))-
{mathcal P}(1, H_1(b), H_2(b),dots, H_n(b)) . $$
Здесь $ H_1(lambda), H_2(lambda),dots, H_n(lambda) $ —
главные миноры матрицы $ A-lambda, E $, а $ {mathcal P}_{} $ — число знакопостоянств.

Доказательство



ЗДЕСЬ.

Согласно этой теореме, главные миноры матрицы $ A-lambda, E $ играют роль системы
полиномов Штурма для характеристического полинома симметричной матрицы $ A_{} $.

=>

Если все главные миноры $ A_1,A_2,dots,A_{n} $ симметричной матрицы $ A_{} $ отличны от нуля, то
число положительных собственных чисел матрицы $ A_{} $ равно числу знакопостоянств, а число отрицательных собственных чисел — числу знакоперемен в ряду $ 1,A_1,dots,A_n $:

$$ operatorname{nrr} { det (A-lambda E) =0 | lambda>0 } = {mathcal P}(1,A_1,dots,A_n),
$$
$$
operatorname{nrr} { det (A-lambda E) =0 | lambda<0 }={mathcal V}(1,A_1,dots,A_n) .
$$

П

Пример. Локализовать собственные числа матрицы

$$
left(
begin{array}{rrr}
11 & 2 & -8 \
2 & 2 & 10 \
-8 & 10 & 5
end{array}
right)
$$

Решение.
$$ H_1(lambda)=11- lambda, H_2(lambda)=lambda^2-13, lambda+18,
$$
$$
f(lambda)= H_3(lambda)=-lambda^3+18, lambda^2 +81, lambda -1458
.
$$

$ lambda $ $ 1_{} $ $ H_1(lambda) $ $ H_2(lambda) $ $ H_3(lambda) $ $ {mathcal P} $ Комментарии
$ 0_{} $ $ + $ $ + $ $ + $ $ — $ 2 число положительных =2
$ -10 $ $ + $ $ + $ $ + $ $ + $ 3 собственное число
$ -5 $ $ + $ $ + $ $ + $ $ — $ 2 лежит на $ ]-10,-5[ $
$ 5 $ $ + $ $ + $ $ — $ $ — $ 2 собственное число
$ 10 $ $ + $ $ + $ $ — $ $ + $ 1 лежит на $ ]5,10[ $
$ 15 $ $ + $ $ — $ $ — $ $ + $ 1 собственное число
$ 20 $ $ + $ $ — $ $ + $ $ — $ 0 лежит на $ ]15,20[ $

Проверка. Спектр матрицы: $ {-9,9,18 } $.

П

Пример. Локализовать собственные числа матрицы

$$
left(
begin{array}{rrr}
1 & -2 & 2 \
-2 & -2 & 4 \
2 & 4 & -2
end{array}
right) .
$$

Решение.
$$H_1(lambda)=1- lambda, H_2(lambda)=lambda^2+, lambda-6,
f(lambda)=H_3(lambda)=-lambda^3-3, lambda^2 +24, lambda -28
.
$$

$ lambda_{} $ $ 1_{} $ $ H_1(lambda) $ $ H_2(lambda) $ $ H_3(lambda) $ $ {mathcal P} $ Комментарии
$ 0_{} $ $ + $ $ + $ $ — $ $ — $ 2 число положительных =2
$ -8 $ $ + $ $ + $ $ + $ $ + $ 3 собственное число
$ -6 $ $ + $ $ + $ $ + $ $ — $ 2 лежит на $ ]-8,-6[ $
$ 1.5 $ $ + $ $ — $ $ — $ $ — $ 2 два собственных числа
$ 3_{} $ $ + $ $ — $ $ + $ $ — $ 0 лежат на $ ]1.5,3[ $

Никаким дроблением интервала $ ]1.5, , , 3[ $ не удается отделить
два вещественных собственных числа. Вывод: имеется кратное собственное
число.

Проверка. Спектр матрицы: $ {-7,2,2 } $.

Произвольная матрица

Собственный вектор

Собственный вектор матрицы 8) $ A_{} $, принадлежащий (или соответствующий) ее собственному собственному числу $ lambda_{ast}^{} $, определяется как ненулевой столбец
$$
X_{ast}= left(
begin{array}{c} x_{1}^{ast} \ vdots \ x_{n}^{ast}
end{array} right)
in mathbb{C}^n
$$
такой, что
$$ AX_{ast}=lambda_{ast} X_{ast} quad iff quad (A -lambda_{ast}E) X_{ast} = mathbb O_{ntimes 1} . $$
По определению собственного числа, $ det (A^{} -lambda_{ast}E) = 0 $ и, следовательно,
система однородных уравнений $ (A -lambda_{ast}E) X^{} = mathbb O $ всегда имеет нетривиальное решение; более того, этих решений бесконечно много. Таким образом, одному и тому же собственному числу матрицы принадлежит бесконечное множество собственных векторов. Эту бесконечность можно описать с помощью фундаментальной системы решений (ФСР).

Строго говоря, только что введено определение правого собственного вектора матрицы $ A $. Потому как для
этой же матрицы определяется и левый собственный вектор — как строка $ Y_{ast}=(y_1^{ast},dots, y_n^{ast}) ne mathbb O $, такая, что
$$ Y_{ast} A= mu_{ast} A quad mbox{ при некотором } mu_{ast} in mathbb C , . $$
Очевидно, что $ Y_{ast} $ является левым собственным вектором $ A $ тогда и только тогда, когда столбец $ Y_{ast}^{top} $ является правым собственным вектором матрицы $ A^{^{top}} $. Кроме того, поскольку спектры матриц $ A $ и $ A^{^{top}} $ совпадают, то все результаты, полученные для правых собственных векторов, автоматически переносятся и на левые. Традиционно принято рассматривать правые собственные векторы; в этом случае слово «правый» опускают.

П

Пример. Найти собственные векторы матрицы

$$
A=
left(begin{array}{rrrr}
0&1&2&3\
-1&0&4&7\
-2&-4&0&2\
-3&-7&-2&0
end{array}right).
$$

Решение. Спектр матрицы найден выше.
$$(A-0 cdot E)X=mathbb O quad Longrightarrow mbox{ ФСР}=
left{
{mathfrak X}_1=left(begin{array}{r}
4 \ -2 \ 1 \ 0 end{array}right),
{mathfrak X}_2=left(begin{array}{r}
7 \ -3 \ 0 \ 1 end{array} right) right}.$$
Любой вектор вида $ alpha_{1} {mathfrak X}_1 + alpha_2 {mathfrak X}_2 $ будет собственным,
принадлежащим $ lambda_{}=0 $.
$$ begin{array}{c}
(A- mathbf i, sqrt {83} E)X=mathbb O \ \ Downarrow \ \ {mathfrak X}_3=
left(begin{array}{c}
1- mathbf i , sqrt {83} \ 8-2, mathbf i , sqrt {83} \ 12 \ 17+mathbf i , sqrt {83}
end{array}right)
end{array}
qquad
begin{array}{c}
(A+mathbf i sqrt {83} E)X=mathbb O \ \ Downarrow \ \ {mathfrak X}_4=
left(begin{array}{c}
1+mathbf i , sqrt {83} \ 8+2mathbf i , sqrt {83} \ 12 \ 17- mathbf i ,sqrt {83}
end{array}right)
end{array} .
$$



Еще один способ нахождения собственного вектора основан на теореме Гамильтона-Кэли.

Т

Теорема 15. Пусть $ lambda_{ast}^{} $ — собственное число матрицы $ A_{} $. Обозначим частное от деления характеристического полинома на линейный множитель $ lambda_{} — lambda_{ast} $ через $ f_{ast}(lambda)^{} $:

$$ f_{ast}(lambda) equiv f(lambda) / (lambda-lambda_{ast}) . $$
Тогда любой ненулевой столбец матрицы $ f_{ast}(A)^{} $ является собственным вектором, принадлежащим $ lambda_{ast}^{} $.

Доказательство следует из равенства
$$(A-lambda_{ast} E)f_{ast}(A)=mathbb O_{ntimes n} . $$
На основании определения любой ненулевой столбец $ f_{ast}(A)^{} $ должен быть
собственным вектором матрицы $ A_{} $.


П

Пример. Найти собственные векторы матрицы

$$
A=left( begin{array}{rrr}
9 & 22 & -6 \ -1 &-4 & 1 \ 8 & 16 & -5
end{array}
right) .
$$

Решение.
$$ det (A-lambda E)=-lambda^3+ 7, lambda + 6 equiv -(lambda_{}-3) (lambda+2)(lambda+1) , .$$
Пренебрегая знаком – , имеем:
$$
begin{matrix}
f_1(lambda)=lambda^2+3lambda+2 & u & f_1(A)=
left( begin{array}{rrr}
40 & 80 & -20 \ 0 &0 & 0 \ 40 & 80 & -20
end{array}
right) , \
f_2(lambda)=lambda^2-2lambda-3 & u & f_2(A)=
left( begin{array}{rrr}
-10 & -30 & 10 \ 5 &15 & -5 \ 0 & 0 & 0
end{array}
right) , \
f_3(lambda)=lambda^2-lambda-6 & u & f_3(A)=
left( begin{array}{rrr}
-4 & -8 & 4 \ 4 & 8 & -4 \ 8 & 16 & -8
end{array}
right) .
end{matrix}
$$

Ответ. $ {mathfrak X}_{1}=[1,0,1]^{^{top}} $ принадлежит $ lambda_{1}^{}=3 $,
$ {mathfrak X}_{2}=[-2,1,0]^{^{top}} $ принадлежит $ lambda_{2}^{}=-2 $,
$ {mathfrak X}_{3}=[-1,1,2]^{^{top}} $ принадлежит $ lambda_{3}^{}=-1 $.

=>

Если $ lambda_{ast}^{} $ является простым корнем характеристического полинома9), то ненулевые столбцы $ f_{ast}(A)^{} $ будут пропорциональными. Или, что то же, $ operatorname{rank} f_{ast}(A)^{} = 1 $.

Тогда очевидно, что и строки матрицы $ f_{ast}(A)^{} $ тоже должны быть пропорциональны!

На практике вычисление полинома $ f_{ast}(lambda)^{} $ может быть осуществлено с помощью схемы Хорнера.

П

Пример. Вычислить собственный вектор матрицы

$$
A=left( begin{array}{rrr}
23 & 75 & -92 \ 6 & 74 & 72 \ 37 & -23 & 87
end{array}
right) ,
$$
принадлежащий ее вещественному собственному числу.

Решение. Характеристический полином
$$ f(lambda)= -lambda^3+184,lambda^2-14751,lambda+611404 $$
имеет единственное вещественное собственное число $ lambda_{ast} approx 96.8817 $. Составляем схему Хорнера
$$
begin{array}{c|cccc}
& -1 & 184 & -14751 & 611404 \
hline
96.8817 & -1 & 87.1183 & -6310.8310 & -0.0352
end{array}
$$
За счет ошибок округления мы получили ненулевое значение для $ f(lambda_{ast}) $. В качестве частного от деления $ f(lambda) $ на $ lambda-lambda_{ast} $ берем
$$
f_{ast}(lambda)= -lambda^2 + 87.1183, lambda — 6310.8310 .
$$
Подставляем в него матрицу $ A_{} $ и вычисляем первый столбец матрицы
$$ -A^2+87.1183,A -6310, E =
left( begin{array}{rrr}
-1882.1101 & * & * \ -2723.2902 & * & * \ -708.6229 & * & *
end{array}
right) .$$
Проверяем:
$$
left( begin{array}{rrr}
23 & 75 & -92 \ 6 & 74 & 72 \ 37 & -23 & 87
end{array}
right)
left( begin{array}{r}
-1882.1101 \ -2723.2902 \ -708.6229
end{array}
right) — 96.8817 left( begin{array}{r}
-1882.1101 \ -2723.2902 \ -708.6229
end{array}
right)= left( begin{array}{r}
0.0356 \ 0 \ -0.0002
end{array}
right) .
$$



Можно развить последний метод далее: найти универсальную формулу для собственного вектора как функции ее собственного числа. Действительно, найдем частное от деления характеристического полинома
$$ f(lambda) =a_0lambda^n+a_0lambda^{n-1}+dots+ a_n, quad a_0=(-1)^n $$
на линейный полином $ lambda- lambda_{ast} $, где $ lambda_{ast} $ — произвольное число из $ mathbb C $. С помощью той же схемы Хорнера, получаем
$$ q(lambda)= $$
$$
=a_0lambda^{n-1}+(a_0lambda_{ast}+a_1)lambda^{n-2}+(a_0lambda_{ast}^2+a_1lambda_{ast}+a_2)lambda^{n-3}+dots+ (a_0lambda_{ast}^{n-1}+a_1lambda_{ast}^{n-2}+dots+a_{n-1}) , .
$$
Если $ lambda_{ast} $ является собственным числом матрицы $ A_{} $, то любой ненулевой столбец матрицы
$$ q(A)= $$
$$
=a_0A^{n-1}+(a_0lambda_{ast}+a_1)A^{n-2}+(a_0lambda_{ast}^2+a_1lambda_{ast}+a_2)A^{n-3}+dots+ (a_0lambda_{ast}^{n-1}+a_1lambda_{ast}^{n-2}+dots+a_{n-1})E
$$
будет собственным вектором, принадлежащим $ lambda_{ast} $.

П

Пример. Найти представление всех собственных векторов матрицы

$$
A=left( begin{array}{rrr}
9 & 22 & -6 \ -1 &-4 & 1 \ 8 & 16 & -5
end{array}
right)
$$
в виде функции ее собственных чисел.

Решение. Характеристический полином матрицы был вычислен выше: $ f(lambda)=-lambda^3+ 7, lambda + 6 $. Имеем,
$$
q(lambda)=-lambda^2-lambda_{ast}lambda+(7-lambda_{ast}^2)
$$
и
$$
q(A)=-A^2-lambda_{ast}A+(7-lambda_{ast}^2)E=
left(begin{array}{ccc} -lambda_{ast}^2-9lambda_{ast}-4 & -22lambda_{ast}-14 & 6lambda_{ast}+2 \
lambda_{ast}-3 & -lambda_{ast}^2+4lambda_{ast}-3 & -lambda_{ast}+3 \
-8lambda_{ast}-16 & -16lambda_{ast}-32 & -lambda_{ast}^2+5lambda_{ast}+14
end{array} right) , .
$$
Берем произвольный столбец этой матрицы, например, первый:
$$
X_{ast}(lambda_{ast})=
left(begin{array}{c}
-lambda_{ast}^2-9lambda_{ast}-4 \
lambda_{ast}-3 \ -8lambda_{ast}-16
end{array} right) , .
$$
Утверждается, что $ X_{ast} (lambda_{ast}) $ — универсальное представление всех собственных векторов матрицы. Действительно,
$$
X_{ast}(-1)
=
left(begin{array}{r}
4 \
-4 \
-8
end{array} right),
X_{ast}(-2)
=
left(begin{array}{r}
10 \
-5 \ 0
end{array} right),
X_{ast}(3)
=
left(begin{array}{r}
-40 \
0 \
-40
end{array} right) , .
$$



Матрица $ q(A) $, которую мы построили для нахождения универсального представления собственного вектора, может быть получена другим способом. В самом деле, поскольку
$$ f(lambda)equiv q(lambda)(lambda-lambda_{ast})+ f(lambda_{ast}), $$
то имеем справедливость матричного равенства:
$$ f(A)equiv q(A)(A-lambda_{ast}E)+ f(lambda_{ast})E , . $$
Откуда, на основании теоремы Гамильтона-Кэли, получаем равенство
$$ q(A)(A-lambda_{ast}E) = — E det (A-lambda_{ast}E) , . $$
Это равенство означает, что матрица $ (- q(A)) $ является взаимной матрице $ A-lambda_{ast}E $:
$$ — q(A)=operatorname{adj}(A-lambda_{ast}E) , . $$
Следовательно, ее выражение (а нам, собственно, нужно только выражение для какого-то ее столбца)
может быть получено с помощью алгебраических дополнений элементов матрицы $ A-lambda_{ast}E $. Именно такой способ вычисления взаимной матрицы использовался при доказательстве теоремы Гамильтона-Кэли.

Т

Теорема 16. Пусть $ g(x)=b_0x^m+dots+b_{m} in {mathbb C}[x] $ – произвольный полином. Если $ X_{ast}in mathbb C^{n} $ — собственный вектор матрицы $ A_{} $,
соответствующий собственному числу $ lambda_{ast}^{} $, то он же будет собственным и для матрицы $ g(A)^{} $, принадлежащим собственному числу $ g(lambda_{ast})^{} $.

Доказательство. Домножим равенство $ A{mathfrak X}_{ast}=lambda_{ast}^{}{mathfrak X}_{ast} $ слева
на матрицу $ A_{} $:
$$ A^2{mathfrak X}_{ast}=lambda_{ast}A{mathfrak X}_{ast}=lambda_{ast}^2{mathfrak X}_{ast} .$$
По индукции доказывается и общее равенство:
$$ A^k{mathfrak X}_{ast}=lambda_{ast}^k{mathfrak X}_{ast} .$$
Домножим его на $ b_{m-k}^{} $ и просуммируем по $ k_{} $ от $ 0_{} $ до $ m_{} $:
$$ g(A){mathfrak X}_{ast}=g(lambda_{ast}){mathfrak X}_{ast} ,$$
что и доказывает утверждение теоремы.


=>

Если матрица $ A $ невырождена, то теорема остается справедливой и для произвольного полинома от $ A^{-1} $. В частности, собственные векторы $ A^{-1} $ совпадают с собственными векторами матрицы $ A $.

Т

Теорема 17. Собственные векторы, принадлежащие различным собственным числам матрицы $ A_{} $, линейно независимы.

Т

Теорема 18. Собственные векторы, принадлежащие различным собственным числам вещественной симметричной матрицы $ A_{} $, ортогональны, т.е. если $ mathfrak X_1 $ принадлежит собственному числу $ lambda_{1} $, а $ mathfrak X_2 $ принадлежит собственному числу $ lambda_{2} $ и $ lambda_1 ne lambda_2 $, то

$$ langle mathfrak X_1, mathfrak X_2 rangle =0 , $$
где $ langle , rangle $ означает скалярное произведение, определяемое стандартным образом: $ langle X,Y rangle =x_1y_1+dots+x_ny_n $.

Доказательство



ЗДЕСЬ.

Теорема Перрона-Фробениуса

Т

Теорема 19 [Перрон, Фробениус]. Для положительной матрицы $ A_{} $ существует положительное собственное число $ lambda_{+} $ такое, что все остальные собственные числа этой матрицы меньше $ lambda_{+} $ по абсолютной величине (модулю). Соответствующий этому собственному числу собственный вектор может быть выбран положительным:

$$ exists mathfrak X_{+} > mathbb O: quad A mathfrak X_{+} = lambda_{+} mathfrak X_{+} . $$

Число $ lambda_{+} $ из теоремы называется собственным числом Перрона или собственным числом Перрона-Фробениуса матрицы $ A_{} $, а соответствующий ему произвольный положительный собственный вектор —
собственным вектором Перрона-Фробениуса матрицы $ A_{} $.

=>

Спектральный радиус положительной матрицы $ A_{} $ совпадает с ее собственным числом Перрона-Фробениуса:

$$ rho(A)=lambda_{+} . $$

П

Пример. Найти собственное число и вектор Перрона-Фробениуса для матрицы

$$
A=
left(begin{array}{rrrr}
2 & 7 & 18 & 28 \
1 & 8 & 2 & 8 \
3 & 1 & 4 & 1 \
5 & 9 & 26 & 5
end{array}
right) , .
$$

Решение. Характеристический полином матрицы $ A_{} $
$$
det(A-lambda E)=lambda^4-19, lambda^3-175, lambda^2-285, lambda+10390
$$
имеет корнями
$$
lambda_{1,2} approx -6.260463 pm 5.452465 mathbf i, lambda_3 approx 5.878976, lambda_4 approx 25.641950 .
$$
Числом Перрона-Фробениуса является $ lambda_4 $, а соответствующий ему собственный вектор Перрона-Фробениуса можно взять равным
$$
left(
begin{array}{c}
1 \
0.365240 \ 0.184802 \ 0.634244
end{array}
right) quad mbox{ или } quad
left(
begin{array}{c}
2.737922 \ 1 \ 0.505974 \ 1.736510
end{array}
right) quad mbox{ или }
left(
begin{array}{c}
5.411185 \ 1.976383 \ 1 \ 3.432010
end{array}
right) quad mbox{ или } quad
left(
begin{array}{c}
1.576681 \ 0.575868 \ 0.291374 \ 1
end{array}
right) quad mbox{ или } quad
left(
begin{array}{r}
0.798133 \
0.291510 \
0.147496\
0.506210
end{array}
right) quad mbox{ или } dots
$$
(напоминаю: собственный вектор определяется с точностью до ненулевого сомножителя!). Последний вектор имеет длину равную $ 1_{} $.




Свойства.


1.

Собственное число Перрона-Фробениуса всегда простое для характеристического полинома матрицы $ A_{} $. Отсюда следует, что собственный вектор Перрона-Фробениуса определяется единственным образом — с точностью до домножения на положительный скаляр.


2.

Любой собственный вектор положительной матрицы $ A_{} $, не соответствующий собственному числу Перрона-Фробениуса, не может состоять исключительно только из положительных элементов. Иными словами, хотя бы одна компонента такого вектора должна быть либо отрицательной либо мнимой.


3.

Для собственного числа Перрона-Фробениуса справедливо неравенство
$$ min_{jin{1,dots,n}} sum_{k=1}^n a_{jk} le lambda_{+} le max_{jin{1,dots,n}} sum_{k=1}^n a_{jk} . $$


4.

Собственное число Перрона-Фробениуса матрицы $ A_{} $ совпадает с собственным числом Перрона-Фробениуса матрицы $ A^{top} $.


Какие из перечисленных свойств можно распространить на случай неотрицательных матриц ? Каждую такую матрицу можно рассматривать как предел последовательности (строго)
положительных матриц.
Воспользовавшись теоремой о непрерывной зависимости собственных чисел матрицы от ее элементов, можем сделать вывод, о том, что для неотрицательной матрицы $ A_{} $ всегда найдется вещественное неотрицательное собственное число, которое будет являться максимальным по модулю среди всех собственных чисел матрицы. Другое дело, что в данном случае — в отличие от случая положительных матриц — такое мажорирующее собственное число может оказаться не единственным.

П

Пример. Спектр неотрицательной матрицы

$$ A=left( begin{array}{cc} 0 & 1 \ 1 & 0 end{array} right) $$
состоит из чисел $ lambda_1=+1 $ и $ lambda_1=-1 $ одинакового модуля.


Однако, по-прежнему, хотя бы одно неотрицательное вещественное число $ lambda_{+} $ со свойством $ rho(A) = lambda_{+} $ существовать будет; более того, ему будет соответствовать неотрицательный собственный вектор $ mathfrak X ge mathbb O $. Это число (вектор) по-прежнему называются числом (вектором) Перрона-Фробениуса10) матрицы $ A_{} $.

Частным случаем неотрицательных матриц являются стохастические матрицы, т.е. неотрицательные матрицы, в которых сумма элементов
каждой строки равна $ 1_{} $:
$$
mathfrak P=left[p_{jk}right]_{j,k=1}^n, {p_{jk}ge 0 }_{j,k=1}^n, sum_{k=1}^n p_{jk} = 1 npu quad j in {1,2,dots,n} .
$$

Т

Теорема 20. Собственное число Перрона-Фробениуса стохастической матрицы равно $ 1_{} $. Этому собственному числу соответствует собственный вектор $ X=[1,1,dots,1]^{top} $.

Доказательство существования собственного числа равного $ 1_{} $ и соответствующего ему собственного вектора $ X=[1,1,dots,1]^{top} $ следует из равенства
$$mathfrak P left(
begin{array}{c}
1 \
1 \ vdots \ 1
end{array}
right) =
left(
begin{array}{c}
1 \ 1 \ vdots \ 1
end{array}right) .
$$
Далее, из теоремы Гершгорина следует, что любое собственное
число $ lambda_{}in mathbb C $ стохастической матрицы должно удовлетворять неравенству
$$|lambda — p_{jj}|le sum_{kne j} |p_{jk}|=1-p_{jj} $$
хотя бы при одном $ j_{} $. Воспользовавшись следствием к неравенству треугольника получаем:
$$|lambda| — |p_{jj}|le |lambda — p_{jj}| le 1-p_{jj} Rightarrow
|lambda| le 1 . $$



Численное нахождение собственного числа и собственного вектора Перрона-Фробениуса возможно по методу, разобранному в пункте



ЧАСТИЧНАЯ ПРОБЛЕМА СОБСТВЕННЫХ ЧИСЕЛ.

Методы вычисления характеристического полинома

Вычисление коэффициентов характеристического полинома матрицы $ A_{} $
непосредственным разложением определителя $ det (A-lambda_{} E) $ на $ n!_{} $ слагаемых — крайне неэффективно. Элементами этого разложения являются выражения, полиномиально зависящие от параметра $ lambda_{} $. На каждом этапе вычислений мы получаем проблему символьных вычислений: хранения таких полиномов и действий над ними.

Основной метод вычисления числовых определителей — метод Гаусса — также неэффективен в приложении к вычислению определителя, элементы которого зависят от параметра.
Источником вычислительных проблем является неудобное расположение переменной $ lambda_{} $ — на главной диагонали матрицы. Первый же шаг метода Гаусса приводит к делению на элемент $ a_{11} — lambda $, и, в дальнейшем, элементы преобразованной матрицы будут уже не полиномами, а рациональными функциями относительно $ lambda_{} $. Следующие шаги метода приводят к возрастанию степеней знаменателей. Необходимость в организации хранения рациональных функций и программировании действий с ними кажется тем более неоправданной, если вспомнить, что окончательный ответ — выражение для $ det (A-lambda_{} E) $ — должно быть полиномом по $ lambda_{} $; т.е. знаменатели дробей в конечном ответе сократятся.

А в качестве усугубляющего положение обстоятельства «на заднем плане» маячит проблема точности вычислений коэффициентов характеристического полинома — чувствительность его корней к возмущению его коэффициентов бывает весьма высокой.

Какой выход предлагается? — Предварительно преобразовать определитель $ det (A-lambda_{} E) $ к виду, когда переменная $ lambda_{} $ оказывается «выметенной» с диагонали на крайний ряд (в столбец или в строку). При этом допускается увеличение размеров (порядка) определителя. Такое представление дает возможность разложения определителя по этому исключительному ряду, и, тем самым, позволяет свести задачу к вычислению числовых
определителей — а уж для этой задачи применение метода Гаусса вполне эффективно.

Метод Леверье

Метод основан на формуле (см. следствие к теореме $ 7 $



ЗДЕСЬ ):
$$ operatorname{Sp} (A^k)=lambda_1^k+dots+lambda_n^k=s_k , $$
т.е. след $ k_{} $-й степени матрицы $ A_{} $ равен $ k_{} $-й сумме Ньютона ее характеристического полинома $ f(lambda)=det (A-lambda E ) $.
Вычисляем последовательные степени матрицы $ A_{} $:
$$s_1=operatorname{Sp} (A), s_2=operatorname{Sp} (A^2), dots, s_n=operatorname{Sp} (A^n) .$$
Неизвестные коэффициенты $ f(lambda)=(-1)^n(lambda^n+a_1lambda^{n-1}+
dots+a_n) $ находим по рекурсивным формулам Ньютона:
$$
a_1=-s_1, a_2=-(s_2+a_1s_1)/2,
$$
$$
a_k=-(s_{k}+a_1s_{k-1}+a_2s_{k-2}+dots+a_{k-1}s_1)/k npu k le n.
$$
Очевидно, что не имеет смысла вычислять все элементы матрицы $ A^{n} $ — достаточно обойтись лишь элементами ее главной диагонали.

П

Пример [Леверье]. Найти характеристический полином матрицы

$$
A=left(begin{array}{rrrr}
-5.509882&1.870086&0.422908&0.008814 \
0.287865&-11.811654&5.711900&0.058717 \
0.049099&4.308033&-12.970687&0.229326 \
0.006235&0.269851&1.397369&-17.596207
end{array}
right) .
$$

Решение.
$$
A^2=left(begin{array}{rrrr}
30.91795128&-30.56848188&2.878480155&0.0031325713\
-4.705449283&164.6764010&-141.3504639&-0.4143169528\
0.3341843103&-106.6094396&193.1869924& -6.756396001\
0.0022236138&-1.904168948&-41.16923134& 309.9628536
end{array}
right),
$$
$$
A^3=left(begin{array}{rrrr}
-179.0125092&431.2849919&-198.8601505& -0.9173897610\
66.38829278&-2562.954533& 2771.458834& -15.49709921\
-23.08728044&2090.291485&-3124.010318& 156.9329019\
-0.649145142&-71.21907809&956.2502143& -5463.723497
end{array}
right),
$$
$$
A^4=left(begin{array}{cccc}
1100.720103& ast& ast& ast \
ast& 42332.23816& ast& ast \
ast& ast& 52669.62534& ast \
ast& ast& ast& 96355.91518
end{array}
right) .
$$
Вычисляем следы матриц:
$$s_1=-47.888430, s_2=698.7441983, s_3=-11329.70086, s_4= 192458.4988 ,$$
и по формулам Ньютона получаем:
$$a_1= 47.888430, a_2 = 797.278764_{displaystyle 8}
, a_3 = 5349.45551_{displaystyle 3},
a_4 = 12296.550_{displaystyle 68} . $$



После нахождения коэффициентов характеристического полинома можно найти его корни каким-либо11) методом. Если $ lambda_{ast}^{} $ — одно
из собственных чисел, то для нахождения соответствующего собственного
вектора воспользуемся алгоритмом из



ПУНКТА. Предположив дополнительно, что это собственное число простое12), обозначим
$$
f_{ast}(lambda)= f(lambda)/(lambda-lambda_{ast})=(-1)^n(lambda^{n-1}
+p_1lambda^{n-2}+dots+p_{n-2}lambda+p_{n-1})
$$
т.е. частное от деления $ f(lambda_{}) $ на $ lambda-lambda_{ast} $. Тогда любой ненулевой столбец матрицы $ f_{ast}(A)^{} $ будет собственным вектором, принадлежащим $ lambda_{ast}^{} $. Как правило, собственным вектором можно взять комбинацию

Степени матрицы $ A_{} $ уже нами посчитаны при вычислении коэффициентов характеристического полинома.

П

Пример. Для приведенного выше примера находим собственные числа:

$$
lambda_1=-17.86326,
lambda_2=-17.15242, lambda_3=-7.57404,
lambda_4= -5.29869 .
$$
Коэффициенты $ f_1(lambda) $ можно определить по схеме Хорнера:
$$
begin{array}{r|r|r|r|r|r}
&1 & 47.888430 & 797.2787648 & 5349.455513 & 12296.55068 \ hline
-17.86326 & 1 & underbrace{30.025170}_{p_{_1}}&
underbrace{260.9313465}_{p_{_2}} &underbrace{688.371028}_{p_{_3}}&
approx 0 \
end{array}
$$
Собственным вектором, принадлежащим $ lambda_{1} $, будет
$$left[ -0.0256_{displaystyle 67},
0.21938_{displaystyle 0},
-0.24187_{displaystyle 1},
1.044526 right]^{^{top}} .$$



Т

Теорема 21. Характеристический полином явно выражается через суммы Ньютона с помощью следующего представления:

$$
f(lambda)=frac{1}{n!}left|
begin{array}{llllll}
s_1 &1 & & & &\
s_2&s_1& 2 & &mathbb O & \
s_3&s_2&s_1&3& & \
vdots& & & ddots &ddots & \
s_n&s_{n-1}& s_{n-2} & dots &s_1&n \
lambda^n&lambda^{n-1}&lambda^{n-2}& dots &lambda&1
end{array}
right|_{(n+1)times (n+1)} .
$$

Этот определитель имеет порядок больший, чем исходный $ det (A_{}-lambda E) $, однако в нем все включения переменной $ lambda_{} $ локализованы в одной строке — именно такое размещение трактовалось как «хорошее» в смысле вычисления определителя



ВЫШЕ.

И

Биографические заметки о Леверье



ЗДЕСЬ.

Существует модификация метода Леверье, позволяющая организовать одновременное вычисление как самого характеристического полинома матрицы $ A_{} $, так и матрицы взаимной к матрице $ A_{}-lambda E $ (что делает возможным получение универсальной формулы для всех собственных векторов матрицы $ A_{} $); этот метод известен как метод Леверье-Фаддеева.

Метод Крылова

Рассмотрим произвольный ненулевой столбец $ Y_0=left[ y_{1}^{[0]},dots,y_{n}^{[0]} right]^{^{top}} in mathbb C^n $. Cоставим итерационную векторную
последовательность
$$
Y_1=Acdot Y_0, Y_2=Acdot Y_1, dots, Y_{n}=Acdot Y_{n-1} .
$$

Т

Теорема 22. Определитель

$$
det left[begin{array}{c|c|c|c|c}
Y_0&Y_{1}&dots&Y_{n-1}&Y_{n}\
1& lambda&dots&lambda^{n-1}&lambda^n
end{array}
right]_{(n+1)times (n+1)}
$$
совпадает — с точностью до постоянного множителя — с характеристическим полиномом матрицы $ A_{} $. Здесь $ |_{} $ означает конкатенацию.

Доказательство. Легко видеть, что
$$ Y_K=A^KY_0 quad npu quad K in {1,dots,n} . $$
Если
$$ f(lambda)=det(A-lambda E) =(-1)^n lambda^n+a_1 lambda^{n-1}+a_2 lambda^{n-2}+dots+a_n , $$
то по теореме Гамильтона-Кэли:
$$
(-1)^n A^n+a_1A^{n-1}+dots+a_nE=mathbb O_{n times n} .
$$
Это равенство останется справедливым и после умножения его на произвольный вектор, в том числе на $ Y_{0} $:
$$
(-1)^n A^ncdot Y_0+a_1A^{n-1} cdot Y_0 +dots+a_ncdot Y_0=mathbb O_{n times 1} iff
$$
$$
iff quad (-1)^n Y_n+a_1Y_{n-1} +dots+a_nY_0=mathbb O .
$$
Последнее равенство представляет линейную систему относительно неизвестных коэффициентов характеристического полинома. Можно решать ее по формулам Крамера, но мы пойдем другим путем. Дополним эту систему тождеством $ f(lambda)=(-1)^n lambda^n+a_1 lambda^{n-1}+a_2 lambda^{n-2}+dots+a_n $. Рассмотрим получившуюся систему как линейную однородную относительно столбца $ left[ a_n,a_{n-1},dots,a_1,1right]^{top} $. Поскольку эта система имеет нетривиальное решение, то ее определитель должен равняться нулю:
$$
0=det left[begin{array}{c|c|c|c|c}
Y_0&Y_{1}&dots&Y_{n-1}&(-1)^nY_{n}\
1& lambda&dots&lambda^{n-1}&(-1)^nlambda^n-f(lambda)
end{array}
right]=
$$
(представляем последний столбец в виде суммы двух столбцов и используем свойство

5

определителя)
$$
=det left[begin{array}{c|c|c|c|c}
Y_0&Y_{1}&dots&Y_{n-1}&(-1)^nY_{n}\
1& lambda&dots&lambda^{n-1}&(-1)^nlambda^n
end{array}
right]-f(lambda)
det left[begin{array}{c|c|c|c}
Y_0&Y_{1}&dots&Y_{n-1}
end{array}
right] .
$$
Таким образом,
$$ f(lambda)=(-1)^n frac{det left[begin{array}{c|c|c|c|c}
Y_0&Y_{1}&dots&Y_{n-1}&Y_{n}\
1& lambda&dots&lambda^{n-1}&lambda^n
end{array}
right]}{det left[begin{array}{c|c|c|c}
Y_0&Y_{1}&dots&Y_{n-1}
end{array}
right]} ,
$$
если только знаменатель в этой дроби не обратится в нуль.


П

Пример. Найти характеристический полином матрицы примера Леверье

$$
A=left(begin{array}{rrrr}
-5.509882&1.870086&0.422908&0.008814 \
0.287865&-11.811654&5.711900&0.058717 \
0.049099&4.308033&-12.970687&0.229326 \
0.006235&0.269851&1.397369&-17.596207
end{array}
right) .
$$

Решение. Возьмем $ Y_0=left[ 1,0,0,0 right]^{top} $. Имеем
$$
begin{array}{cccc}
Y_1=A Y_0= & Y_2=AY_1= & Y_3=AY_2= & Y_4=AY_3= \
left(begin{array}{r}
-5.509882\
0.287865 \
0.049099 \
0.006235
end{array}
right), &
left(begin{array}{r}
30.917951\
-4.705449 \
0.334184 \
0.002223
end{array}
right), &
left(begin{array}{r}
-179.012509\
66.388293 \
-23.087280\
-0.649145
end{array}
right), &
left(begin{array}{r}
1100.720101\
-967.597333\
576.522644\
-4.040153
end{array}
right) .
end{array}
$$
$$
det left[begin{array}{c|c|c|c|c}
Y_0&Y_{1}&Y_2& Y_{3}& Y_{4}\
1& lambda&lambda^2 &lambda^{3}&lambda^4
end{array}
right]=
left| begin{array}{rrrrr}
1 & -5.509882 & 30.917951 & -179.012509 & 1100.720101 \
0 & 0.287865 & -4.705449 & 66.388293 & -967.597333\
0 & 0.049099 & 0.334184 & -23.087280 & 576.522644\
0 & 0.006235 & 0.002223 & -0.649145 & -4.040153 \
1 & lambda & lambda^2 & lambda^3 & lambda^4
end{array}
right|=
$$
$$
=0.348621 lambda^4+16.694915lambda^3+277.948166lambda^2+1864.932835lambda+4286.836454 =
$$
$$
=0.348621 left(lambda^4+47.888430lambda^3+797.27876_{displaystyle 3}lambda^2+5349.4555_{displaystyle 0}lambda+12296.550_{displaystyle 5} right) .
$$



После нахождения характеристического полинома можно найти его корни каким-либо13) методом. Пусть $ lambda_{ast}^{} $ — одно из собственных чисел, и оно —
простое; тогда для нахождения соответствующего собственного вектора можно воспользоваться тем же приемом, что был задействован в предыдущем ПУНКТЕ. Вычислим14) частное от деления $ f(lambda_{}) $ на $ lambda-lambda_{ast} $
$$
f_{ast}(lambda)= f(lambda)/(lambda-lambda_{ast})=(-1)^n(lambda^{n-1}
+p_1lambda^{n-2}+dots+p_{n-2}lambda+p_{n-1}) .
$$
Тогда любой ненулевой столбец матрицы $ f_{ast}(A)^{} $ будет собственным вектором, принадлежащим $ lambda_{ast}^{} $. Но тогда и произвольная комбинация столбцов этой матрицы тоже будет собственным вектором (если только не обратится в нулевой вектор). В частности, это относится и к комбинации, записываемой в матричном виде
$$ (-1)^n f_{ast}(A) Y_0 = A^{n-1}Y_0 +p_1A^{n-2}Y_0+dots+p_{n-1}Y_0=Y_{n-1}+p_1Y_{n-2}+dots+p_{n-1}Y_0 . $$
А комбинируемые векторы уже посчитаны.

Теперь обсудим исключительные случаи. При неудачном выборе $ Y_{0} $ определитель
$$
det left[begin{array}{c|c|c|c}
Y_0&Y_{1}&dots&Y_{n-1}
end{array}
right]
$$
может обратиться в нуль. Эта неприятность обязательно произойдет если, например, наш выбор пал на вектор $ Y_0 $, совпадающий с собственным вектором матрицы $ A_{} $. Вероятность такого события — нулевая. В общем же случае, трудно ожидать, чтобы $ n_{} $ почти произвольных столбцов
$ Y_0,Y_{1},dots,Y_{n-1} $ оказались линейно зависимыми — если только сама матрица $ A_{} $ не обладает «скрытым дефектом» — типа рассмотренного в следующем примере.

П

Пример. Найти характеристический полином матрицы

$$A=left( begin{array}{ccc}
2&1&1 \
1&2&1 \
1&1&2
end{array}
right) .
$$

Решение. При любом выборе $ Y_0 $ векторы $ {Y_0,Y_1,Y_2 } $ оказываются линейно зависимыми:
$$
Y_0=
left(begin{array}{r}
1\
0\
0
end{array}
right),
Y_1=
left(begin{array}{r}
2\
1\
1
end{array}
right),
Y_2=
left(begin{array}{r}
6\
5\
5
end{array}
right),dots ;
Y_0=
left(begin{array}{r}
1\
1\
1
end{array}
right),
Y_1=
left(begin{array}{r}
4\
4\
4
end{array}
right),dots
$$
Объяснение этого феномена состоит в том, что для матрицы $ A_{} $ ее аннулирующий полином имеет степень меньшую ее порядка:
$$ A^2-5 A+4 E = mathbb O . $$
Домножение этого равенства на произвольный столбец $ Y_0 $ и доказывает линейную зависимость системы $ {Y_0,Y_1,Y_2} $.


Такая ситуация возможна только в случае, когда характеристический полином матрицы $ A_{} $ имеет кратные корни (в рассмотренном выше примере $ lambda_{}=1 $ являлся двойным корнем $ det (A-lambda_{} E) $); она исключительно редко встречается на практике.

Поиск всех собственных чисел

Существуют методы нахождения спектра матрицы, не требующие предварительного построения характеристического полинома.

QR-алгоритм

Этот алгоритм основан на QR-разложении матрицы $ A $.

Т

Теорема 23. Спектр матрицы $ A $ совпадает со спектром матрицы $ P^{top} A P $ при произвольной ортогональной матрице $ P $.

Доказательство.
$$ det (P^{top} A P-lambda E)=det (P^{top} A P- lambda P^{top} E P)=det P^{top} (A -lambda E ) P =
det (A -lambda E ) P P^{top} = det (A -lambda E ) , .
$$



Пусть QR-разложение матрицы $ A $ имеет вид
$$ A=Q_1R_1 , , $$
где $ Q_1 $ — ортогональная, а $ R_1 $ — верхнетреугольная матрицы. Тогда матрица
$$ A_2=R_1Q_1 $$
имеет тот же спектр, что и матрица $ A $. Действительно, поскольку
$$ A_2=Q_1^{top} A Q_1 , $$
то сработает предыдущая теорема. Вычислим QR-разложение матрицы $ A_2 $
$$ A_2=Q_2R_2 $$
и переставим местами матрицы этого произведения:
$$ A_3=R_2Q_2 , . $$
Матрица
$$ A_3= Q_2^{top} A_2 Q_2=Q_2^{top} Q_1^{top} A Q_1 Q_2 $$
продолжаем иметь те же собственные числа, что и матрица $ A $. Утверждается, что бесконечная последовательность матриц
$$ {A_j=R_{j-1}Q_{j-1}}_{j=1}^{infty} $$
как правило, сходится к матрице $ A_{infty} $, которая будет верхнетреугольной.

Т

Теорема 24 [4]. Если все собственные числа матрицы $ A $ различны по модулю, то матрица $ A_{infty} $ является верхнетреугольной и на ее главной диагонали стоят собственные числа матрицы $ A $.

П

Пример. Найти все собственные числа матрицы

$$
A=left(begin{array}{rrr}
2 & 3 &-1\
7 & 3 & 3 \
-1 & -2 & 4
end{array}
right) , .
$$

Решение.
$$
A_1=Aapprox
underbrace{left(begin{array}{rrr}
0.272165 & 0.759752 & 0.590511 \
0.952579 & -0.299517 & -0.053683 \
-0.136083& -0.577119 & 0.805242
end{array}
right)}_{Q_1}
underbrace{left(begin{array}{rrr}
7.348469 & 3.946400 & 2.041241\
0 & 2.534941 & -3.966781 \
0 & 0 & 2.469409
end{array}
right)}_{R_1}
$$
Теперь переставляем матрицы произведения местами и строим QR-разложение получившейся матрицы:
$$
quad Rightarrow quad A_2 = R_1Q_1approx
left(begin{array}{rrr}
5.481481 & 3.222957 & 5.771191 \
2.954542 & 1.530046 & -3.3303021 \
-0.336044 & -1.425143 & 1.988472
end{array}
right)approx
$$
$$
approxunderbrace{left(begin{array}{rrr}
-0.878992 & 0.022595 & 0.476300\
0.473781 & -0.154267 & -0.867026 \
0.053886 & -0.987771 & 0.146304
end{array}
right)}_{Q_2}
underbrace{left(begin{array}{rrr}
-6.236096& -3.634658 & -3.387848\
0 & 1.244502 & -1.319999\
0 & 0 & 5.927198
end{array}
right)}_{R_2}
$$
Продолжим процесс:
$$
quad Rightarrow quad A_3 = R_2Q_2approx
left(begin{array}{rrr}
7.020952& 3.766220 & -0.314568\
-0.660752 & 1.111870 & -1.272137\
0.319398 & -5.854713 & 0.867177
end{array}
right) approx
$$
$$
approx
underbrace{left(begin{array}{rrr}
-0.994581 & -0.065879 & 0.080426 \
0.093601 & -0.230749 & 0.968501 \
-0.045246 & 0.970780 & 0.235665
end{array}
right)}_{Q_3}
underbrace{left(begin{array}{rrr}
-7.059205 & -3.376839 & 0.154554 \
0 & -6.188319 & 1.156106 \
0 & 0 & -1.053002
end{array}
right)}_{R_3}
$$
Замечаем тенденцию убывания элементов матриц $ {A_j} $, стоящих под главной диагональю.
$$
Rightarrow dots Rightarrow A_{10} approx
left(begin{array}{rrr}
mathbf{6.}_{246022} & 2.758769 & -2.160057\
-0.0467437 & mathbf{4.4}_{09292} & -5.341014\
0.000018 &-0.005924 & mathbf{-1.6}_{55314}
end{array}
right) approx
$$
$$
underbrace{left(begin{array}{rrr}
-0.999972 & -0.007483 & 0.000007 \
0.007483 & -0.999971 & 0.001339 \
-0.000003 & 0.001339 & 0.999999
end{array}
right)}_{Q_{10}}
underbrace{left(begin{array}{rrr}
-6.246197 & -2.725694 & 2.120031\
0 & -4.429817 & 5.354807 \
0 & 0 & -1.662479
end{array}
right)}_{R_{10}} , .
$$
Матрица $ Q_j $ уже близка к диагональной (с элементами $ pm 1 $), верхнетреугольность матрицы $ A_j $ также заметна, но точность приближения еще не достаточна.
$$
Rightarrow dots Rightarrow A_{20} approx
left(begin{array}{rrr}
mathbf{6.17}_{5608} & 2.805821 & -2.020513 \
-0.001776 & mathbf{4.48}_{4917} & -5.388407\
0 & 0 & -mathbf{1.660525}
end{array}
right) approx
$$
Точность приближения минимильного собственного числа существенно выше точностей приближения остальных чисел.
$$
Rightarrow dots Rightarrow A_{30} approx
left(begin{array}{rrr}
mathbf{6.172}_{778} & 2.807524 & -2.015076\
-0.000073 & mathbf{4.487}_{747} & -5.390442\
0 & 0 & -mathbf{1.660525}
end{array}
right) , .
$$



К сожалению условие теоремы достаточно ограничительно: собственные числа вещественной матрицы $ A $ могут оказаться и мнимыми, но тогда они одинаковы по модулю.

§

Как это обстоятельство сказывается на структуре матрицы $ A_{infty} $ и дальнейшее развитие метода



ЗДЕСЬ

Частичная проблема собственных чисел

Задача. Найти максимальное по модулю собственное число матрицы $ A_{} $.


Предположение

. Будем считать сначала, что максимальное по модулю собственное число матрицы единственно.

Излагаемый ниже метод поиска этого собственного числа называется методом степенны́х итераций15).

Рассмотрим произвольный ненулевой столбец $ Y_0=left[ y_{1}^{[0]},dots,y_{n}^{[0]} right]^{^{top}} in mathbb C^n $. Cоставим такую же итерационную векторную
последовательность, как и в методе Крылова
$$
Y_1=Acdot Y_0, Y_2=Acdot Y_1, dots, Y_{K}=Acdot Y_{K-1},dots ,
$$
(только теперь, в отличие от метода Крылова, считаем ее неограниченно продолжающейся) и выделим последовательность первых элементов этих векторов:
$$y_{1}^{[1]},y_{1}^{[2]},dots,y_{1}^{[K]},dots $$

Т

Теорема 25. Как правило, предел

$$
lim_{Kto +infty}frac{y_{1}^{[K+1]}}{y_{1}^{[K]}}
$$
существует и он равен максимальному по модулю собственному числу матрицы $ A_{} $.

Доказательство. Перенумеруем собственные числа $ lambda_{1},dots,lambda_n $ матрицы $ A_{} $ так, чтобы $ lambda_{1} $ обозначило максимальное по модулю:
$$|lambda_1|= max_{jin {1,dots,n}} |lambda_j| , quad |lambda_1|>|lambda_j| quad npu quad jin {2,dots,n} . $$
Очевидно,
$$
Y_{K}=A^Kcdot Y_0 ;
$$
отсюда следует, что любой элемент столбца $ Y_{K} $ может быть линейно выражен через $ lambda_{1}^K,dots,lambda_n^K $.
В частности, это справедливо и для первого элемента:
$$
y_{1}^{[K]}=C_1lambda_1^K+C_2lambda_2^K+dots+C_nlambda_n^K .
$$
В этом представлении $ {C_j}_{j=1}^n $ — будут константами из $ mathbb C_{} $ в случае если все собственные числа являются простыми, и полиномами из $ mathbb C[K] $ в случае, если имеются кратные собственные числа. Действительно, в первом случае существует базис пространства $ mathbb C^n $, состоящий из собственных векторов матрицы $ A_{} $:
$$ A{mathfrak X}_j=lambda_j{mathfrak X}_j quad npu quad jin {1,dots,n} . $$
Вектор $ Y_0 $ можно разложить по этому базису:
$$Y_0=alpha_1{mathfrak X}_1+dots+alpha_n{mathfrak X}_n .$$
Тогда последовательным домножением на матрицу $ A_{} $ получаем :
$$begin{matrix}
Y_1=AY_0&=& alpha_1 lambda_1{mathfrak X}_1+dots+alpha_nlambda_n{mathfrak X}_n, \
dots & & dots \
Y_K=A^KY_0&=& alpha_1 lambda_1^K{mathfrak X}_1+dots+alpha_nlambda_n^K{mathfrak X}_n
end{matrix}
$$
откуда и следует доказываемое равенство.

Во втором случае — когда имеются кратные собственные числа матрицы $ A_{} $ — придется применять «тяжелую артиллерию» в виде жордановой нормальной формы; см. теорему $ 5 $



ЗДЕСЬ. Для простоты рассуждений, будем в оставшейся части доказательства считать все собственные числа матрицы различными. Имеем тогда
$$
lim_{K to +infty} frac{y_{1}^{[K+1]}}{y_{1}^{[K]}}=
lim_{K to +infty} frac{lambda_1^{K+1} left[C_1+
C_2(lambda_2/lambda_1)^{K+1}+dots+
C_n(lambda_n/lambda_1)^{K+1} right]}
{lambda_1^{K} left[C_1+C_2(lambda_2/lambda_1)^{K}+dots+
C_n(lambda_n/lambda_1)^{K} right]}
=lambda_1
$$
поскольку
$$
lim_{K to +infty} left| frac{lambda_j}{lambda_1} right|^K = 0 quad
npu quad jin {2,dots,n} .
$$
Исключительным случаем является ситуация $ C_1=0 $, в этом случае утверждение теоремы может оказаться несправедливым16).


=>

Как правило, вектор

$$
left[1, lim_{Kto +infty}frac{y_{2}^{[K]}}{y_{1}^{[K]}},dots,
lim_{Kto +infty}frac{y_{n}^{[K]}}{y_{1}^{[K]}}right]^{^{top}}
$$
будет собственным, принадлежащим максимальному по модулю собственному числу матрицы $ A_{} $.

П

Пример. Для матрицы

$$
A=left(begin{array}{rrr}
2 & 3 &-1\
7 & 3 & 3 \
-1 & -2 & -4
end{array}
right)
$$
найти максимальное по модулю собственное число и принадлежащий ему собственный вектор.

Решение. Возьмем в качестве стартового столбца $ Y_0=[1,0,0]^{^{top}} $. Имеем:
$$
Y_1=AY_0=left( begin{array}{r} 2 \ 7 \ -1 end{array} right),
Y_2=AY_1=left( begin{array}{r} 26 \ 32 \ -12 end{array} right),
Y_3=AY_2=left( begin{array}{r} 160 \ 242 \ -42 end{array} right),dots,
$$
$$
Y_{19}=left( begin{array}{r}
{scriptstyle 4259667747238636} \ {scriptstyle 6435097324667832} \ {scriptstyle -1571397155909260}
end{array} right), Y_{20}=AY_{19}=left( begin{array}{r}
{scriptstyle 29396024624390028} \ {scriptstyle 44408774736946168} \ {scriptstyle -10844273772937260}
end{array} right)
$$
Смотрим на отношения первых элементов векторов:
$$
begin{array}{c|c|c|c|c|c|c|c|c|c}
K & 1 & 2 & 3 & 4 & 5 & dots & 15 & dots & 19 \
hline
y_{1}^{[K+1]}/y_{1}^{[K]} & 2 & 13 & 6.153846 & 6.8 & 7.180147 & dots & 6.900726 & dots & mathbf{6.90101}_{displaystyle 3}
end{array}
$$
Далее, в соответствии со следствием, собственный вектор, принадлежащий найденному числу
$$
approx left[1, frac{y_{2}^{[20]}}{y_{1}^{[20]}},frac{y_{3}^{[20]}}{y_{1}^{[20]}}right]^{^{top}} approx left[1, 1.51070_{displaystyle 6}, -0.368902 right]^{^{top}}
$$



Результат теоремы представляет собой обобщение метода Бернулли поиска максимального по модулю корня полинома. Если в качестве матрицы $ A_{} $ взять матрицу Фробениуса
$$
mathfrak F=
left( begin{array}{lllllll}
0 & 1 & 0 & 0 & dots & 0 & 0 \
0 & 0 & 1 & 0 & dots & 0 & 0 \
0 & 0 & 0 & 1 & dots & 0 & 0 \
vdots& &&&ddots & & vdots \
0 & 0 & 0 & 0 & dots & 0 & 1 \
a_n & a_{n-1} & a_{n-2} & & dots & a_2 & a_1
end{array} right)_{n times n} ,
$$
то равенство
$$ X_K=mathfrak F X_{K-1} npu Kin {1,2,dots } $$
определяет — при задании (произвольным образом) чисел $ x_0,x_1,dots,x_{n-1} $ — линейную рекуррентную последовательность порядка $ n_{} $:
$$
x_{n+K}=a_1 x_{n+K-1}+ dots+ a_n x_K .
$$
Здесь столбцы $ X_K $ определяются формулами
$$
X_0=left( begin{array}{l}
x_0 \ x_1 \ vdots \ x_{n-1}
end{array}
right), X_1=left( begin{array}{l}
x_1 \ x_2 \ vdots \ x_{n}
end{array}
right), X_2=left( begin{array}{l}
x_2 \ x_3 \ vdots \ x_{n+1}
end{array}
right), dots,
X_K=left( begin{array}{l}
x_K \ x_{K+1} \ vdots \ x_{K+n-1}
end{array}
right),dots ;
$$
Характеристический полином последовательности совпадает с характеристическим полиномом матрицы $ mathfrak F $, т.е. с $ lambda^n-a_1 lambda^{n-1} -a_2 lambda^{n-2} -dots — a_n $.

Метод степенных итераций используется в поисковике Google для вычисления PageRank.


Теперь обсудим исключительные случаи алгоритма.



1.


Нарушение сходимости итерационного процесса за счет неудачного выбора стартового вектора. Если в качестве $ Y_{0} $ оказался случайно взят собственный вектор $ mathfrak X_{ast} $ матрицы $ A_{} $, принадлежащий произвольному ее собственному числу $ lambda_{*} $, то предел последовательности из теоремы будет равен именно этому числу; если при этом $ |lambda_{*} | ne max_{1le j le n} | lambda_j | $, то мы выйдем за пределы смысла выражения «как правило». Понятно, что вероятность настолько плохого выбора нулевая, но и выбор $ Y_0 $ вблизи $ mathfrak X_{ast} $ также может существенно замедлить скорость сходимости. Поэтому если возникает ситуация медленной «стабилизации» значащих цифр в десятичном приближении собственного числа, попробуйте сменить начальный вектор.



2.

Нарушение условия

предположения

,
выдвинутого в начале пункта: максимальное по модулю собственное число неединственно.

П

Пример. Найти максимальное по модулю собственное число матрицы примера Леверье

$$
A=left(begin{array}{rrrr}
-5.509882&1.870086&0.422908&0.008814 \
0.287865&-11.811654&5.711900&0.058717 \
0.049099&4.308033&-12.970687&0.229326 \
0.006235&0.269851&1.397369&-17.596207
end{array}
right) .
$$

Решение. Для столбца $ Y_0=[1,0,0,0]^{^{top}} $ имеем
$$y_{1}^{[100]}/y_{1}^{[99]}=-17.8_{displaystyle 3113} ,$$
т.е. на $ 100 $-й итерации получаем лишь $ 3_{} $ истинные десятичные цифры в представлении собственного числа. При этом
компонентами векторов $ Y_{K} $ являются числа порядка $ 10^{123} $. Если мы
посмотрим на ответ примера Леверье, то увидим, что имеются два собственных числа матрицы, близких по модулю.


К сожалению, вероятность того факта, что у случайно выбранной матрицы два ее собственных числа будут иметь одинаковый модуль становится ненулевой если эта матрица выбирается из множества вещественных матриц. Дело в том, что в этом случае ее характеристический полином будет иметь вещественные коэффициенты, а мнимые корни такого полинома всегда пáрные — для любого невещественного корня $ lambda_{ast}^{} $ полинома, комплексно сопряженное к нему число $ overline{lambda_{ast}} $ также будет корнем. При этом $ |lambda_{ast}|= |overline{lambda_{ast}} | $.

П

Пример. Для матрицы

$$
A=left(begin{array}{rrr}
3 & 2 &7\
-2 & -8 & 2 \
5 & -3 & -2
end{array}
right)
$$
итерационная последовательность из теоремы ведет себя хаотически: при выборе $ Y_0=[1,0,0]^{top} $ получим
$$
left{frac{y_{1}^{[K+1]}}{y_{1}^{[K]}}right}_{K=1}^{infty} = 3; 13.3333; 5.9250;
4.6455; 15.9273; 0.8546; 68.0186;
$$
$$
0.4543; 91.7873; dots
$$
Спектр матрицы: $ { 6.9363, -6.9682pm 3.0186 mathbf i} $, и максимальное по модулю собственное число неединственно.


Существуют приемы, позволяющие модифицировать метод на случай когда два числа спектра матрицы близки по модулю к максимальному; они подробно обсуждаются в [3].


Предположение 2

. Пусть два максимальных по модулю собственных числа матрицы разнесены по величине, например
$$ |lambda_1| > | lambda_2 | > | lambda_ j | quad npu j in {2,dots, n } . $$

Обобщение степенного метода основывается на использовании последовательностей из каких-то двух компонент векторов $ Y_{K+1}=AY_K $, например, наряду с уже использованной выше последовательностью первых компонент
$$y_{1}^{[1]},y_{1}^{[2]},dots,y_{1}^{[K]},dots $$
возьмем еще и аналогичную для вторых:
$$y_{2}^{[1]},y_{2}^{[2]},dots,y_{2}^{[K]},dots $$

Т

Теорема 26 [Эйткен]. При практически любом выборе стартового вектора $ Y_0 ne mathbb O $ для последовательности

$$
Y_1=Acdot Y_0, Y_2=Acdot Y_1, dots, Y_{K}=Acdot Y_{K-1},dots ,
$$
имеет место равенство
$$
lambda_1 lambda_2 = lim_{Kto +infty} frac{left|begin{array}{ll} y_1^{[K+1]} & y_1^{[K+2]} \ y_2^{[K+1]} & y_2^{[K+2]} end{array} right|}
{left|begin{array}{ll} y_1^{[K]} & y_1^{[K+1]} \ y_2^{[K]} & y_2^{[K+1]} end{array} right|} , .
$$

Доказательство. Построим квадратное уравнение
$$ p_0x^2+p_1x+p_2 = 0 $$
имеющее корнями $ lambda_1 $ и $ lambda_2 $.
Если существует базис рпостранства $ mathbb C^n $
$$Y_0=alpha_1{mathfrak X}_1+alpha_2{mathfrak X}_2+dots+alpha_n{mathfrak X}_n .$$
Тогда последовательным домножением на матрицу $ A_{} $ получаем :
$$begin{array}{llll}
Y_K=& alpha_1 lambda_1^K{mathfrak X}_1 &+alpha_2 lambda_2^K{mathfrak X}_2+dots &+alpha_nlambda_n^K{mathfrak X}_n, \
Y_{K+1}=& alpha_1 lambda_1^{K+1}{mathfrak X}_1 &+alpha_2 lambda_2^{K+1}{mathfrak X}_2+dots &+alpha_nlambda_n^{K+1}{mathfrak X}_n,\
Y_{K+2}=& alpha_1 lambda_1^{K+2}{mathfrak X}_1 & +alpha_2 lambda_2^{K+2}{mathfrak X}_2+dots &+alpha_nlambda_n^{K+2}{mathfrak X}_n.
end{array}
$$
Отбрасываем из правых частей равенств слагаемые порядков возрастания ниже, чем $ lambda_2^K, lambda_2^{K+1}, lambda_2^{K+2} $ соответственно, домножаем получившиеся приближенные равенства
$$begin{array}{lll|l}
Y_K & approx alpha_1 lambda_1^K{mathfrak X}_1 &+alpha_2 lambda_2^K{mathfrak X}_2, & color{Red} times p_2 \
Y_{K+1}& approx alpha_1 lambda_1^{K+1}{mathfrak X}_1 &+alpha_2 lambda_2^{K+1}{mathfrak X}_2, & color{Red} times p_1\
Y_{K+2} & approx alpha_1 lambda_1^{K+2}{mathfrak X}_1 & +alpha_2 lambda_2^{K+2}{mathfrak X}_2, & color{Red} times p_0
end{array}
$$
и складываем:
$$ p_2 Y_K + p_1Y_{K+1} + p_0 Y_{K+2} approx mathbb O , . $$
В получившемся векторном равенстве выбираем первые две компоненты:
$$
left{
begin{array}{ll}
p_2 y_1^{[K]} + p_1 y_1^{[K+1]} + p_0 y_1^{[K+2]} approx 0 , , \
p_2 y_2^{[K]} + p_1 y_2^{[K+1]} + p_0 y_2^{[K+2]} approx 0 , ,
end{array}
right.
$$
которые и позволят определить приближенное значение набора $ p_0,p_1,p_2 $. С точностью до числового сомножителя, искомый полином можно представить в виде определителя
$$
p_0x^2+p_1x+p_2 approx
left|begin{array}{lll}
y_1^{[K]} & y_1^{[K+1]} & y_1^{[K+2]} \
y_2^{[K]} & y_2^{[K+1]} & y_2^{[K+2]} \
1 & x & x^2
end{array}
right| , .
$$
Формулы Виета завершат доказательство.


=>

При выполнении условия

предположения 2

имеет место равенство

$$ lambda_2 =
lim_{Kto +infty} frac{left|begin{array}{ll} y_1^{[K+1]} & y_1^{[K+2]} \ y_2^{[K+1]} & y_2^{[K+2]} end{array} right| y_{1}^{[K+1]}}
{left|begin{array}{ll} y_1^{[K]} & y_1^{[K+1]} \ y_2^{[K]} & y_2^{[K+1]} end{array} right| y_{1}^{[K+2]} } , .
$$

П

Пример. Для матрицы

$$
A=left(begin{array}{rrr}
2 & 3 &-1\
7 & 3 & 3 \
-1 & -2 & 4
end{array}
right)
$$
найти первые два по порядку убывания модулей собственных числа.

Решение. Для $ Y_0= [1,0,0]^{top} $ имеем:
$$
Y_1=left( begin{array}{r} 2 \ 7 \ -1 end{array} right),
Y_2=left( begin{array}{r} 26 \ 32 \ -20 end{array} right), dots,
$$
$$
Y_{18}=left( begin{array}{r} {scriptstyle 164983395620948} \ {scriptstyle 156537857759336} \ -{scriptstyle 219402049179956} end{array} right),
Y_{19}=left( begin{array}{r} {scriptstyle 1018982413699860} \ {scriptstyle 966291195084776} \ -{scriptstyle 1355667307859444} end{array} right),
Y_{20}=left( begin{array}{r} {scriptstyle 6292505720513492} \ {scriptstyle 5964748557575016} \ -{scriptstyle 8374234035307188} end{array} right), .
$$
Таким образом,
$$
lambda_1 approx frac{y_1^{[20]}}{y_1^{[19]}}= frac{1573126430128373}{254745603424965} approx mathbf{6.17}_5 ,
$$
$$
lambda_2 approx
frac{left|begin{array}{ll} y_1^{[19]} & y_1^{[20]} \ y_2^{[19]} & y_2^{[20]} end{array} right| y_{1}^{[19]}}
{left|begin{array}{ll} y_1^{[18]} & y_1^{[19]} \ y_2^{[18]} & y_2^{[19]} end{array} right| y_{1}^{[20]} }=
frac{300892177677465751832368453246033765185}{67074186846991590001957412392957971737 }
approx mathbf{4.48}_5
$$



Обобщение этого подхода для поиска следующих по убыванию модулей собственных чисел проводится по аналогии с деления-вычитания вычисления корня полинома. Кстати, теоретически можно довести этот метод и до построения характеристического (точнее, минимального аннулирующего) полинома матрицы: при любом $ K in {0,1,2,dots } $ определитель
$$
left|begin{array}{llll}
y_1^{[K]} & y_1^{[K+1]} & dots & y_1^{[K+n]} \
y_2^{[K]} & y_2^{[K+1]} & dots & y_2^{[K+n]} \
vdots & vdots & & vdots \
y_n^{[K]} & y_n^{[K+1]} & dots & y_n^{[K+n]} \
1 & lambda & dots & lambda^n
end{array}
right|=
det left[begin{array}{c|c|c|c|c}
Y_K&Y_{K+1}&dots&Y_{K+n-1}&Y_{K+n}\
1& lambda&dots&lambda^{n-1}&lambda^n
end{array}
right]
$$
совпадает — с точностью до числового сомножителя — с $ det (A-lambda E) $. И мы вернулись к



методу Крылова вычисления характеристического полинома!

Для случая симметричной матрицы альтернативное обобщение степенного метода для поиска ранжированных по убыванию модулей собственных чисел и соответствующих им собственных векторов разбирается



ЗДЕСЬ.

Задачи

Источники

[1]. Островский А.М. Решение уравнений и систем уравнений. М. ИЛ, 1963, c. 137-142

[2]. Уилкинсон Дж.Х. Алгебраическая проблема собственных значений. М.Наука. 1970, с.93-94

[3]. Фаддеев Д.К., Фаддеева В.Н. Вычислительные методы линейной алгебры. М.ГИФМЛ. 1960

[4]. Хорн Р., Джонсон Ч. Матричный анализ. М.Мир.1989

Понравилась статья? Поделить с друзьями:
  • Как составить протокол осмотра участка местности
  • Как найти загс по адресу прописки
  • Как найти сережки на улице
  • Как исправить датчик положения дроссельной заслонки
  • Как правильно составить своего персонажа